Bca 2
Bca 2
1
Dear Students,
We, at CIMAGE, are dedicated to provide the best of education and to
bring an overall positive growth in our students. No wonder, that’s the
reason, our students have been Toppers in the University Examinations;
and after successful completion of the course, they have been provided
with the best placements or have been selected in top ranking institutes for
further studies. I am happy that CIMAGE have been pioneer in setting
milestone in the segment of higher education, for others to be followed.
The publication of ‘CIMAGE Courseware’ is also one of the several
innovative initiatives taken by our college. This textbook is unique in several ways. It has been
written in simple language, so that students could understand even the complex and tough
topics very easily. Every chapter has been divided into three sections, ‘The Objective’, ‘The
Chapter itself’ & ‘The Summary’ part. The ‘Objective’ section will help the students to understand
about the ‘Objective and Purpose’ of the chapter. It will also help them in understanding, how to
use the knowledge, studied in the chapter. The second section is ‘The Chapter’ itself, which has
the relevant study material, prescribed by the university. In this section some practical examples
have also been cited to clarify concept. The third section is the ‘Summary of the chapter’. It has
been written using Hindi & English both the languages. Hence it will help the student from Hindi
medium background also to understand the concept Properly, Deeply and Completely. At the
end of the chapters, relevant Projects, Assignments and Questions have also been given, to
make the students well prepared in the studied chapter.
I am confident that this book will serve the purpose of providing a Clear and Concise guide-line
to the students. However, there is ‘No limit’ of acquiring the knowledge, hence I advice the
students to read other text & reference books as well, as it will help them in gaining more
knowledge in the subject. At CIMAGE Library, a number of Reference Books, Video Lectures,
Journals, Research Papers are there to serve this purpose.
I suggest the students, not to ‘Mug-up’, but to ‘Understand’ the subject properly. You should
attend the classes seriously and if something is not well understood by you; Do not hesitate in
asking questions from your teachers. I also suggest students to discuss the topic and subjects in
a group, as group study has its own advantages. Group Study makes the study easy and
interesting, at the same time, it also makes you aware of the different prospective and
dimensions.
You Know, at CIMAGE, we have one more innovative system for you, i.e. ‘Teacher – Student
Doubt Clearing Session’. If you have any doubts in any chapter, you may meet the respective
teachers individually and get your doubts cleared. Every teacher has been assigned a special
time table for it. You are advised to get benefited with it.
At last, but not the least, I thank all the faculty members of CIMAGE for their valuable
contribution in preparing the courseware.
If you have any suggestions or concerns, you may mail me at [email protected]. Go
ahead; Study Well, Give your Best & Keep adding glory to the rich legacy of CIMAGE.
Thank You.
Neeraj Agrawal
Director,
CIMAGE Group of Institutions.
2
TABLE OF CONTENTS
Chapter-1
BASIC CONCEPTS OF SET THEORY 1-28
Chapter-2
BINARY OPERATIONS & FUNDAMENTAL
LAWS OF ALGEBRA 29-68
Chapter-3
MATRICES 69-136
Chapter-4
LINEAR PROGRAMMING 137-166
3
CHAPTER - 1
Objectives
Unit
Basic concepts of set theory
Carterian product of set
Relation
Equivalence relation, mapping, types of mapping, inverse
mapping, countable set uncountable set
INTRODUCTION
Students are well acquainted with the basic concepts of “Set theory”, so, we
give below a brief description of sets and related concepts.
The notion of a set was first explicitly introduced and developed by G. Cantor
(1845-1918). Cantorian set theory is also called „naive‟ set theory.
1
“distinguishable” we mean one must be able to determine whether two objects of the
set are different or the same.
Intuitively, a set is a well-defined collection of objects.
By „well defined‟ we mean that it is possible in principle to determine whether
an object is a member of a set or not.
Examples of set:
(i) Set of all Asian countries.
(ii) Set of natural numbers between 1 and 25.
(iii) The set of the solutions of x2 - 5x + 6 = 0.
(iv) Set of real numbers etc.
Notation of a set and its elements:
Generally we denote sets by capital letters of English alphabet and its members
by small letters.
If x is an element of the set A we express it by witting x A. and read it as „x
belongs to set A‟ or „x is a member of the set A‟.
If y is not a member of the set A we write it as X X or X X,
Representation of a set:
There are two widely used methods of representing a set-one is to list the
members and the other is to state a property which is possessed only by each member
of the set.
The first form is known as the Tabular form in the middle bracket. { }.
For example, A = {a, e, i, o, u},
N = { 1, 2, 3, …….}, etc.
The second form is popularly known as the Set builder form in which we state
the common property possessed by the elements of the set. If P (x) is a property
which is satisfied by each member x of the set A, then we write
A = {x : P (x)} or A = {x } P (x)}.
For example, the set of natural numbers can be written in this form as
2
N = {x : x is a natural number}
= {1, 2, 3, …….} (Tabular form.)
Clearly A = {x : x2 – 5x + 6 = 0} {2, 3}.
Note. One should be cautious while representing a set in set builder form, as
we can imagine such sets which cannot be actually a set. A careless representation
may lead to (set theoretic) paradoxes.
For example, Russell’s Paradox.
Russell observed that if A is a set, then either A A or A A, since a given
object is either a member of a given set or is not a member of that set. In general, a set
is not member of itself. So let us consider the set X of all sets that are not members of
themselves, i.e.
X {x : xis a set and x x}.
both X X and X X.
both X X , and X X.
Here we find that X is not actually a set as there is an object X itself for which it is
neither possible to say that X is a member of X nor X is not a member of X.
3
A
1
2 3
4
Classifications of sets
(i) Finite and Infinite sets: If the process of counting the elements of as set
comes to an end, the set is called a finite set.
For example, (i) the set of the months of a year, (ii) {x : 1 < x < 10 and x is a
natural number} etc.
If the process of counting of elements of a set does not come to an end, the set
is called an infinite set.
For example, (i) N = {1, 2, 3,….}, (ii) x : x is an integer.}
(ii) Null set or Empty set or Void set: For Convenience the concept of null set
has been introduced.
A set having no element is called a null set. This set is denoted by .
For example, {x : x ≠ x}, {x : x2 + 1 = 0} etc, are null sets
(iii) Singleton set : The set having one and only one element is called a
singleton set.
For example, A = {0}, B = {x : x - 4 = 0}.
(iv) Pair set : The set having two and only two elements is called a pair set.
For example,
A = 1, 2}, B = {x : x is a real number and x2 =16} = {+ 4, -4}.
Note. Meaning of symbols , , :
We write the symbol “ x” for “for all x”.
If p and q are two statements, then we write „p and q‟ symbolically as „p q‟.
For example, x Aand x B is written as x A x B .
Similarly „p or q‟ is written as „p q‟.
5
Thus is a B, a A, then A B.
Conversely is A B, then a A a B, a A.
6
It A and B are equal sets, we write A = B.
For example, if A = {2, 3}, B = {x : x2 – 5x + 6 = 0}
= {x : (x – 3) (x – 2) = 0}
= {2,3},
then A = B.
It should be noted that
A B A B and B A.
(Anti-symmetric law)
(iv) If A B and B C, then A C (Transitive law).
7
Power Set
The power set of a set A is a set whose elements are all possible subsets of the
set A.
It is denoted by P (A) or 2A.
It is clear from definition that
X P ( A) X A.
Universal Set
In any discussion the universal set is a set which is the superset of all the sets
of the discussion, i.e. every set of the discussion is a subset of the universal set or
universe of discourse.
It is generally denoted by or U.
As for example, if U = {1, 2, 3, 4, 5, 6},
A = {1, 2, 3, 4, 5, 6}, B = {2, 4, 6}, C = {1, 3, 5,}
Then U is the universal set in context of the sets
A, B, C as A U , B U , C U.
Universal set may vary according to the context.
Disjoint Sets
If two sets are such that they have no element in common, they are said to be
disjoint.
As for example, A = {1, 2, 3,}, B = {4, 5, 6} are disjoint, whereas if C = {2, 4,
7} then A and C are not disjoint.
8
Operations of Union and Intersection of Two Sets
(i) Union of two sets:
The union of two sets A and B is a set whose elements are al those (and only
those) which belong to A or to B or to both,
i.e. A B {x : x Aor x B}
Example : (i) If A = {x: x R and 4 x 7}
B = {x: x R and 5 x 9}
then A B{x : 5 x 7}.
(ii) A U U.
(iii) If A B , then A and B .
(iv) A B when and only when A B B.
Cartesian Product of Sets
The raw notion of a set does not meet the demands of Mathematics.
A more sophisticated concept of a set with members arranged in a certain order
is need. Such objects constructed from a set by ordering the members of the set are of
fundamental important in Mathematics.
Ordered n-tuple
If n be any natural number and if a1, a2,…. an be any objects then the object (a1,
a2,…. an) whose elements are in specified order is called an ordered n-tuple.
If the number of elements in an ordered n-tuple is two, it ordered 2-tuple or an
ordered triplet etc.
9
Ordered pairs are of great importance in Mathematics as we have seen while
studying co-ordinate geometry of two dimensions. Each point of Cartesian plane
represents an ordered pair (a, b) of real numbers.
In three dimensional Eucleadian geometry a point represents an ordered triplet.
Each point of an Argand‟s diagram represents an ordered pair of real numbers to
indicate a complex number.
In the ordered pair (a, b), a is called the first co-ordinate and b is called the
second co-ordinate.
Clearly ordered pair (a, b) (b, a)
and (a, b) (c, d ) iff a c, b d
Cartesian Product
Let A = {1, 2}, B = {3, 4}. Now consider the pairs
(1, 3), (1, 4), (2, 3), (2, 4).
In these pairs the first co-ordinates are the elements of the set A and the second
co-ordinates are those of the set B. The set consisting of the ordered pairs
= {(1, 3), (1, 4), (2, 3), (2, 4)}.
Definition. The Cartesian product of two sets A and B, written as A B (read
as „A cross B‟) is a set whose elements are all possible ordered pairs (a, b) where
a A and b B.
In the set builder form
B A {(b, a) : b B, a A}.
Similarly B A {(b, a) : b B, a A}.
Example. If A = {1, 2, 3}, B = {7, 5}
Then A B = {(1, 7), (1, 5), (2, 7), (2, 5), (3, 7), (3, 5)}.
10
Note. The process of pairing can be best understood with the help of the
following (tree) diagram for A B
B A = {(7, 1), (7, 2), (7, 3), (5, 1), (5, 2), (5, 3)}.
Clearly A B B A as (1,7) (7,1) etc.
It should also be noted here that for any two sets A and B it is not necessary
that A B=B A.
Also it is not essential that A and B should be different sets in A B.
Meaning of A A
The Cartesian product of the set A with itself, denoted by
A A, is the set of all ordered pairs (a, b) of the elements of the set A.
Sol. Here A B = {(1, 2), (1, 3), (1, 4), (2, 2), (2, 3), (2, 4)}
and C D = {(1, 7), (1, 9), (3, 7), (3, 9),}
(A B) (C D) = {(1, 2), (1, 3), (1, 4), (2, 2), (2, 3),
(2, 4), (1, 7), (1, 9), (3, 7), (3, 9),}.
Again A C = {1, 2, 3}
and B D = {2, 3, 4, 7, 9}
(A C) (B D) = {(1, 2), (1, 3), (1, 4), (2, 2), (2, 3),
(2, 4), (1, 7), (1, 9), (3, 7), (3, 9),}
Clearly (A B) (C D) (A C) (B D).
11
1. (b) If A = {a, b}, B = {b, c, d}, and C = {a, c, e, f} than verify
A (B C) ( A B) ( A C).
12
(a, d, a), (a, d, d), (b, b, a), (b, b, d),
(c, b, a), (c, b, d), (c, c, a), (c, c, d),
( c, d, a), (c, d, d)}.
For convenience the so-called tree-diagram is used to find
A B C. The tree-diagram is as follows-
We write the elements of the sets as shown or as the case suggests. Then
proceeding along the lines as arrows indicate we get the elements of the product set.
EXERCISE
1. (i) If A = {2, 3}, B = {4, 5,}, C = {5, 6}, then find
(a) A (B C); (b) A (B C)
1. (ii) If A = {a, b,}, B = {c, d) and C = {e}, then verify the following:
I. (A -B) C = (A C) – (B C)
II. A (B – C) = (A B) – (A C)
III. A (B C) = (A B) (A C).
1. (iii) If A = {0, 3}, B = {4, 5,}, C = {7, 12}, D = {13, 5}
verify that
(a) (A B) (C D) = (A C) (A C) (B D)
2. (i) Prove that (A B) (C B) = when A C= .
(ii) If X = A B prove that
X X = (A A) (B B).
13
(iii) (A’ A) (A B’) (A’ B)‟.
Relation
Definition of Relation (as sets of ordered pairs).
If A and B be two non-empty sets, then every subset R of
A B is a relation from A to B. Thus if R A B, then R is a relation from A to B.
Example. If A = {(1, 2}, B = {a, b), then
R1= {(1, a}, (2, b), R2 = {(1, b}, (2, a), (1, a)}
R8 = {(1, a)} are all relations from A to B.
Definition of Relation (as a propositional function).
If R is a rule such that for every ordered pair (a, b) A B. aRb is a statement
i.e. aRb is either true or false, then R is relation from A to B.
If (a, b) R i.e. aRb then we say a is R-related to b.
Some Definitions. If R is a relation from the set A to the set B then the set
RD = (x | x A, ( x, y) R}
i.e. the set of first co-ordinates of the members of R is called the domain of the
relation.
The set Rr = (y | y B, ( x, y) R} i.e. the set of second co-ordinate of the
members of R is called the range of the relation.
Example. If A = {1, 3, 5, 7 } and B = {2, 4, 6}
Then R = {(1, 2), (1, 4) (3, 6) A B.
So R is a relation from A to B.
Also domain of R = {1, 3}; range of R = {2, 4, 6}.
14
Since A B and A B A B, and A B are both relations from A to
B. is called the empty or void relation and A B is called the universal relation
from A to B.
If R A B then set R-1 = {(b, a)} | (a, b) R} B A is called the inverse
relation of R.
15
i.e. xRy yRx where x, y A.
Example. If A = {1, 2, 3}, then
R1= {(1, 2}, (2, 1), (2, 3), (3, 2), (1, 4) (4, 1)}
is a symmetric relation. But
R2 = {(1, 2}, (2, 1), (2, 3), (1, 4) (4, 1)}
is a symmetric relation
Example. If N is the set of natural numbers and x y x is less than or equal
to y then (N, ) is a well- ordered set because “ ” is partial order relaion in N and
every subset of N has a least member. But under the same relation, (R, ) where R is
the set of real number, is not a well-ordered set for the open interval ]a, b[ R does
not have a least member.
WORKED OUT EXAMPES
1. In the set A = {1, 2, 3} define a relation which is reflexive and
symmetric but not transitive. Give reasons.
Solution. Let R = {(1, 1}, (2, 2), (3, 3), (4, 4), (1, 2) (2, 1), (3, 1), (1, 3)}, R is
a relation on A since R A A.
Again (x, x) R, x A
So R is a reflexive relation.
Again (1, 2) R, (2, 1) R, and (3, 1) R, (1, 3) R,
thus (x, y) R (y, x) R.
So, R is a symmetric relation.
Lastly (3, 1) R, and (1, 2) R, but (3, 2) R,
Thus (x, y) R (y, z) R do not imply (x, z) R,
So, R is not transitive relation.
2. Prove that the relation „=‟ is an equivalence relation in the power set P(A) of
the non-empty set A.
Solution Let X, Y, Z,….. be the subsets of the set A, i.e. X, Y, Z,….. P(A).
(i) X, X, X P(A).
16
So, relation „=‟ is reflexive.
(ii) Again if X = Y then Y = X.
X=Y Y = X. So, the relation „=‟ is symmetric.
(iii) If X = Y and Y = Z then we know X = Z.
X = Y and Y = Z X = Z.
So, the relation „=‟ is transitive also.
The relation „=‟ is an equivalence relation.
17
R ={(1, 2), (1, 4), (2, 3)}
Is a relation as R A B but not a function as 3 A but (3, y) R for any y B and also
(1, 2), (1, 4) R but 2 4.]
Here x is called pre-image or independent variable and y is called image or
dependent variable or value of f.
Domain, Co-domain and Range:
If f : A - B is a mapping, the set A is called the domain of the mapping f and the
set B is called its co-domain. Again the set of images of the elements of A is called the
range of f and is denoted by f(A).
For example, if f : N R where f(x) = x2, then the domain of f =N, co-domain
of f =R and range of f = {1, 4, 9, 16, …….}.
Some special types of functions:
(i) Single-valued function. Since in f . A B every x A associates with
single element of y B, so f is often called single-valued function.
(ii) Real-valued function. Since in f : A R, the function f is called or real-
valued function.
For example, f : R R, where f (x) = x3 +2.
(iii) Constant function.
If the mapping f : A B, is such that every element of A has the same image in
B (i.e. the range of the function is a singleton set), then f is called a constant function
or mapping.
For example, f : R R, where f (x) = 2, is a constant function.
(iv) Equal function. If f : A B, and g : A B be such that f (x) = g(x),
x A, then f and g are said to be equal.
Here domain and range of both the functions are equal.
For example, if f : R R, where f (x) = x2, x R
and g:R R+, where g(y) = y2, y R
then f (x) = g(x).
18
(v) Identity mapping.
If f : A A is such that f (x) = y A i.e. image of every element of A is the element
itself, then f is called an identity mapping or function. For example, If A = {1, 2,
3}and f ={(1, 1), (2, 2), (3, 3)}
then f:A A, is an identity mapping denoted by IA.
19
then f = {(1, a), (2, b), (3, c)}
is an onto mapping. Here f (A) = {a, b, c} = B.
(iii) One-one mapping.
If the mapping f : A B be such that no element of B has more than one pre-
image in the set A, i.e. different elements of A have different images in B, then f is
called an one-one mapping. One-one mapping is also sometimes called injective
mapping.
20
(a) f (x1) = f (x2) x1 = x2 , x1, x2 A.
(b) f (A) B.
For example, if A = {a, b, c, d}, B ={1, 2, 3, 4, 5}
then f = {(a, 1), (b, 2), (c, 3), (d, 4)}
is an one-one into mapping.
21
If f : A B is many-one as well as onto i.e. at least one element of B has more
than one pre-image in A and every element B is the image of some element of A, then
f is called a many-one onto mapping.
Thus f : A B is many-one onto iff
(a) f (x1) = f (x2) does not imply x1 = x2 , x1, x2 A.
(b) f (A) = B.
For example, if A = {a, b, c, d}, B ={1, 2,}
then f = {(a, 1), (b, 1), (C, 2), (d, 2)}
is a many-one onto mapping.
Inverse mapping.
Let f : A B be a mapping. If there exists a rule by which each element y and B can
be associated with a unique element x A where y = f (x), then rule is called the
inverse mapping of f and is written as f-1: B A. x is called the inverse image of y
and is written as f-1(y) = x.
As a set f-1= {(y, x) : x A, y B and (x, y) f}.
It is clear from definition that all mappings have not inverse mappings. We
shall see ahead that only one-one onto mapping has inverse mapping.
Example.
A to B. Let f : A B where A = {1, 2, 3, …}
B = {3, 5, 7, …} and f (x) = 2x + 1
y 1
then y = 2x + 1, or x .
2
y 1
f-1: B A where f 1 ( y) is the inverse of f.
2
COUNTABLE AND UNCOUNTABLE SETS
Introduction. A question that whether tow given sets have the same number of
elements naturally arises. If the sets are finite we can answer the question by counting
22
the elements of the two sets. But if the sets are infinite the answer depends upon how
we define the notion of two sets to have the same number of elements.
The credit goes to Georg Cantor who brought a revolution in the theory of sets
by introducing the concept of numerical equivalence.
Definition of Equivalent sets. If A and B are two sets and there exists an one-one
onto function f: A B, then A is said to be numerically equivalent or simply
equivalent or equipotent to B.
In order words, a set A is said to be equivalent to as set B if there exists an one-
one map of A onto B i.e. there exists an one-one correspondence between A and B,
symbolically A B.
The map (f) is called numerical equivalence between A and B. Numerical
equivalence is denoted by or . Thus if A is numerically equivalent to B, we write A
B
Example. (i) if A = {a, b, c, d}, B = {2, 5, 6, 7} and there exists a map f : A
B defined by
f (a) = 5, f (b) = 2, f (c) = 7, f (d) = 6,
then f is one-one onto. So, A B.
(ii) If A = { 1, 2, 3, …., n, …}
B = { 2, 4, 6, …., 2n, …}
then the map f : f (x) = 2x + 3 is one-one onto. So X Y
Theorem 1. Numerical equivalence of sets is an equivalence relation.
Proof . Here we find that
(i) A A, since there exists the indentity map IA: A A which is one-one
onto. So is reflexive
(ii) If A B, then there exits a map f: A B which is one-one onto and then
f-1: B A is one-oine onto i.e. B A. So A B B A Hence is symmetric.
(iii) If A B, B C, then there exits a map f: A B. g:
23
B C which are one-one onto. Now if
f: A B, g: B C,
then there exists a map gf : A C which is one-one onto i.e. A C. So A B, B C
A C. Hence is transitive.
In this way we see that is an equivalence relation.
Finite set and Infinite set. A set is called finite if it is either void or has an one-one
correspondence with the set of the form {1, 2, ….., n} where n N.
Alternatively, a finite set is also defined as follows. A set is finite if it cannot
be placed in one-one correspondence with (i.e. is not equivalent to) any of its proper
subsets.
A set is infinite if it can be placed in one-one correspondence (is equivalent to)
with at least one of its proper subsets.
Thus if a set is not finite then it is infinite.
Example. (i) If A = {a, b, c, d}, B ={1, 2, 3, 4,} and a 1, a 1, b 2, c 4,
d 3, then A is a finite set and has 4 elements.
(ii) If A = {a, b, c,}, its proper subsets are {a}, {b}, {c}, {a, b}, {b, c},{a, c}.
Here we find that none of these subsets can be placed in one-one
correspondence with A. So, A is finite set.
It should also be noted that any two singleton sets are equivalent.
(iii) Let N = {1, 2, 3,…..}, B ={2, 4, 6,…..} N
Let f: N E be defined by f(x) = 2x.
Clearly f is one-one onto. So, N E.
Thus N is an infinite set.
Similarly Z, Q, R, C etc. all are infinite sets.
Denumerable set. If a set is equivalent to N, the set of natural numbers, then A is
called denumerable (enumerable) set, i.e. N A A is denumerable.
More explicitly, A is denumerable if there exists an one-one onto map of N to A.
This is a contradiction as all the elements of I = [0, 1] were labelled.
24
Thus we conclude that I is uncountable.
WORKED OUT EXAMPLES.
1. Prove that two enumerable sets are equivalent.
Solution. Let X and Y be both enumerable sets.
To prove that X Y.
Since X and Y are enumerable X N, Y N.
Since is symmetric Y N N Y.
Since is transitive,
X N, N Y, X Y.
2. Prove that if A be a non-denumerable set and B be a
denumerable set, then A – B is non-denumerable.
Solution. If possible, let A – B be a denumerable set.
Now, (A – B) (A B) = A.
As A B B and B is denumerable, so A B is denumerable.
As union of two denumerable sets is denumerable, (A – B) (A B) i.e. A is
denumerable which contradicts the fact that A non-denumerable.
Hence A – B must be non-denumerable.
3. Prove that the product set N N is denumerable.
Solution. We know that A B is denumerable, if A and B are denumerable.
Here since N is a denumerable set, so their Casrtesian product N N is
denumerable.
4. Prove that the set of rational numbers is countable (denumerable).
y
Solution. A rational number is of the form where x, y Z.
z
y
Writing as ordered pair (x, y), the set of rational numbers
x
Q = {(x, y) : x Z, y Z}.
25
For a fixed x Z, the set Zx = {(x, v) : y Z} is countable since Z is countable.
Now the set Q = Zx.
But Zx. Being countable union of countable sets is countable.
So, Q is countable.
5. Prove that the set of irrational numbers is uncountable (non-
denumerable).
Solution. Let the set of irrational numbers be countable.
Since the union of two countable sets is countable, so the union of the set of
rational numbers and the set of irrational numbers is countable.
This implies the set of real numbers is countable which contradicts the fact that
the set of real numbers is uncountable.
So, the set of irrational numbers is uncountable.
6. Prove that the set of all algebraic numbers is countable.
Solution. We know that a real number x is called an algebraic number if x is
solution of a polynomial equation
a0 + a1y + a2y2 + …. + anyn = 0,
where a0, a1, a2,..….. are integers.
Let E be the set of all polynomial equations.
For each pair (m, n) of natural numbers, let E(m, n) be the set of polynomials
of degree n such that
| a0 | +| a1| + | a2 | + …. + | an | = m,
Clearly E is a countable union of countable sets.
So, E is countable.
Let E = {P1(x) = 0, P2(x) = 0, P3(x) = 0, ……}.
Let E2 = {Set of solutions of P2(x) = 0}.
As a polynomial of degree n can have n roots, each E2 is finite. But the set A of
algebraic numbers = K.
k N
2
26
EXERCISE
1. Prove that every superset of an uncountable set is uncountable.
2. Construct an example to show that countable union of countable sets is countable.
[Hint. A1= {1, 3, 5, …….}, A2= {1, 4, 7, ……}……
An= {1, n+2, 2n+3….}….
N {2}.]
n 1
3. Prove that the set of (i) integers, (ii) negative integers is countable.
4. If the universal set be a countable set, then prove that the
complementary set of a countable set is countable.
5. If A and B be denumerable sets, then prove that A–B is denumerable.
[Hint. A- B A and A is denumerable.]
6. If B is a countable subset of an uncountable set A, then prove that A – B
is uncountable.
[Hint. Let A – B be countable.
So, (A – B) B is countable.
But B A, (A – B) B=A B = A.
So, A is countable, a contradiction.]
7. Prove that the interval [0, 1] is an uncountable subset of R.
(set theory)
(collection)
B A (subset) B A
B ⊂ A, B, A
27
A B B, A
B A (proper)
(null)
φ
A B A B (union) A
B
intersection A B A B
(intersection) A B
A–B A B
A U, U–A U A (complement)
(function)
X Y
Y’ X
f
2 16
4
4 36
6 49
A B
f:A B
28
CHAPTER - 2
Objectives
Unit
Binary operations
Fundamental laws of algebra
Abelian group, Cyclic group, Subgroup
Ring
Integral domain Field
INTRODUCTION
The readers are already familiar with the set R of real numbers. In this set we
know that corresponding to each ordered pair of real numbers (a, b) there is a unique
real number a + b and a unique real number a b. We say, + (addition) and
(multiplication) are two binary operations on the set of real numbers. They are called
binary because corresponding to two real numbers we get a real number under each of
these operations. Readers also know certain properties of these operations, viz,
a + b = + a for all a, b R
(it is called commutative law of addition),
a b=b a for all a, b R
(it is called commutative law of multiplication),
a + (b+ c) = (a + b) + c for all a, b, c R
(it is called associative law of multiplication) etc.
29
It is the presence of many laws that are satisfied for the operations + and ,
which make the set of real numbers very rich and these give the set of real numbers, a
strong mathematical structure.
This motivates us to think how some mathematical structure can be built on a
set in general. A set in itself does not have a mathematical structure. For that, some
operations are to be defined on the set which will satisfy certain laws. It is a well-
known fact that the disciplines of Physical, Social and Biological sciences are based
on some fundamental laws, established by observations. On the basis of these laws,
the whole structure of the science is erected. In the same way, the laws are satisfied by
any operation over a set, impose mathematical structure on the set.
For example
Let A = {1, 2}
Then A A = {(1, 1), (1, 2), (2, 1), (2, 2)}.
30
(ii) Relation. A subset R of the set A B is called relation from the set A to
the set B. If R is a relation and (a, b), R where a A, b B we say a is R-related to
b and write it as (a, b), R or a Rb.
For example
Let A = {2, 4, 6, 8}
and B = {3, 7}.
Then “<” ( calledless than) is a relation from A to B
and < = {(2, 3), (2, 7), (4, 7), (6, 7)}.
i.e. 2 < 3, 2 < 7, 4 < 7, 6 < 7.
For example
Let A = {2, 4, 6, 8}.
Then “<” is a relation from R
and < = {(2, 4), (2, 6), (2, 8), (4, 6), (4, 8), (6, 8)}.
(iii) Mapping. A mapping f from the set A to the set B, denoted by f: A B,
is a rule which associates each element x A to an uniquely determined element y B.
The set A and B are called domain and co-domain respectively while y is called the
image of x (denoted by y = f (x) or x y) and x is called the pre-image of y under the
mapping f. The set of images of the elements of A, denoted by f (A), is called the
range of the mapping (or function).
If B = A, then f is mapping from A to A.
For example
Let A = {1, 2, 3} and B ={x, y, z, w}.
Then the rule f defined by 1 x, 2 y, 3 z is a mapping of A into the set B.
The mapping f: A B can also be understood as a subset of the Cartesian
product of A and B. A subset f of A B is a mapping of A into the set B if every
element of A appears as first co-ordinate once and only once in the elements of f. The
mapping in the above example can be written as
f = {(1, x), (2, y), (3, x),
31
Obviously mapping f is also a relation, but of a special kind.
There are many kinds of mapping –
(a) Into, Onto mapping. A mapping f: A B is called into mapping if f
(A) B i.e. there is at least one element of B which is not the image of any element of
A. But if f (A) = B, the mapping is called onto mapping.
For example
Let the mapping f: R R, R being the set of real numbers, be defined as
follows –
f (x) = x2.
Clearly f is an into mapping as no negative real number of co-domain R is the
image of any real number of the domain R.
But if f: R R, R is defined as follows –
f (x) = 2x + 1
Then the mapping is onto mapping as every real number y of the co-domain R is the
y 1
image of the real number of the domain R.
2
(b) Many-one, One-one mapping. A mapping f : A B is called many-one
if f (x1) = f (x2) does not imply x1 = x2 for x1 x2 A.
So,for a many-one mapping two distinct elements of A can have the same
image. But if f (x1) = f (x2) x1 = x2 for all x1 x2 A then the mapping is called one-
one.
For example
Let the mapping f: R R, R being the set of real numbers, be defined as
follows –
f (x) = x2.
Clearly f (x) = f (– x) though x – x for all x. Hence the mapping is many-one,
x and – x having the same image.
But if it is defined as f (x) = 2x + 1 then
32
f (x1) = f (x2) 2x2 + 1 = 2x2 + 1
x1 = x2 for all x1 x2 R.
Hence the mapping will be one-one.
A mapping which is both one-one and onto is also called bijective mapping or
one-one correspondence.
An one-one correspondence f: A A, is also called a transformation or
permutation on A.
33
(iv) Ordinary division „ ‟ for real number is a binary operation R – {0} but
not a binary operation over the set Z of integer.
(v) and of sets are binary operation over the set P(A) i.e. power set of
A for union or intersection of two subsets of A (i.e. members of P(A)) is also a subset
of A (i.e. a member of P(A)).
Note. Though varied types of operations can be defined on
a set, usually we shall write +, , o to denote operations.
34
Let X be the set of non-zero real numbers, „x‟ is a binary operation over X.
Also a b=b a for all numbers a and b belonging to X. So the operation „X‟ is
commutative over the set X. But the binary operation over X is not commutative for
a b b a where a, b are any two non-zero real numbers.
(iii) Associative law. Let A be a non-empty set and „o‟ is a binary operation
defined over A such that
a o (b o c) = (a o b) o c for all a, b, c A,
then the binary operation „o‟ is said to be associative over the set A.
For example
Let A be non-empty set and P(A) is the set of all subsets of A (i.e. power set of
A). „ ‟ (union) is a binary operation over P(A) for which we know
A1 (A2 A2) = (A2 A2 ) A2 for all A1, A2, A2 P(A).
Now take the operation „o‟ defined over the set P(A) as follows –
A1 o A2 = A1- A2 (i.e. difference set).
Clearly „o‟ is not associative over P(A) because
(A1 o A2) o A3 A1 o (A2 o A2).
{ (A1 o A2) o A3 = (A1- A2) – A2 and A1 (A1 o A2) A1– (A2 – A2)
and we know form set theory, these are unequal}.
Note. The associative law in more general form is
.. (a o b) o (c o d) = a o ( b o c) o d
= a o (b o c) = (a o b) o c} o d.
(iv) Distributive law. Let A be a non-empty set and „o‟ „*‟ are two binary
operation defined over A such that
a o (b c) = (a o b) (a o c)for all a, b, c A,
then the binary operation „o‟ is said to be associative over in the set A.
If a (b o c) = (a b) o (a c) for all a, b, c A,
then the binary operation „*‟ is distributive over the operation „o‟.
35
Thus, the set A is closed under the binary operation „o‟ if a o b A, for all
a A,
For example
Let A be a non-empty set P(A) is the set of all subsets of A. „ ‟ (union) and
„ ‟ (intersection) are tow binary operations over P(A) . Also we know from Set
theory,
A1 (A2 A3) = (A2 A2 ) (A1 A2) for all A1, A2, A3 P(A), i.e. „ ‟ is
distributive over „ ‟
Also A1 (A2 A3) = (A1 A2 ) (A1 A3 )
for all A1, A2, A3 P(A),
„ ‟ is distributive over „ ‟
For the binary operations + (addition) and (multiplication) over the set R of real
numbers we know
a (b + c) = a b+a c for all a, b, c R
a + (b + c) (a + b) (a + c).
Thus „ ‟ is distributive over „+‟ but „+‟ is not distributive over „ ‟.
(v) Existence law of Identity Element. Let A be a non-empty set and a
binary operation „o‟ „*‟ is defined over A for which a particular element e A exists
such that
aoe=eoa=aa A
then we say that the identity element exists in A for the binary operation „o‟.
For example
For the binary operation „ ‟ (ordinary multiplication) over the set R of real
numbers, the identity element exists and the identity element is 1 because
a 1=1 a = a for all a Z.
For the binary operation „+‟ (ordinary addition) over the set E of even natural
numbers, the identity element does not exists because there is not element a E such
that
36
a + e = e + a for all a E.
Note. If for the binary operation „o‟ over the set A, there exists
an element e such that a o e = a for all a E then e is called the right identity
element. If e o a = a for all a E then e is called the left identity element.
Clearly existence of identity element implies that the right identity element and
the left identity element are equal.
(vi) Existence law of Inverse Element. Let A be a non-empty set and a
binary operation „o‟ is defined over A such that each a A there exists a corresponding
element a’ A such that
a o a’ = a’ o a = e,
Where e is the identity element of A for the operation „o‟ then we say that for the
binary operation „o‟ over the set A, the inverse element exist and a, a’ are called
inverse elements of each other.
For example
For the binary operation „+‟ (ordinary addition) over the set Z of integers,
inverse elements exist because corresponding to each element a Z we have integer –
a A such that
a+ (– a) = (–a) + a = 0,
(the identity element for „+‟ over Z)
But for the binary operation „ ‟ (ordinary multiplication) over the set Z, inverse
elements do not exist because corresponding to each a Z we cannot find an element
a’ Z such that
a a‟ = a‟ a = 1,
(the identity element for „ ‟ over Z)
Note. If for the binary operation „o‟ over the set A, corresponding to a E there
exists a’ A such that
a’ o a = e,
37
(the identity element for „o‟ over A)
then a’ is called the right inverse element of a while if a’ o a = e, we say a’ is
the left inverse element of a. Clearly the existence of right as well as left inverse
elements of a A imply the existence of the inverse element of a A.
Algebraic Structure
A set A is said to have an algebraic structure if at least one binary operation is
defined over A and one or more of the algebraic laws (section 1.4) for the operations
hold. Algebraic structures may be imposed on a set by two or more binary operations.
In the next chapters we study some of the simple algebraic structures, namely
semi-group and group for one binary operation; ring, integral domain and field for
two binary operations. The study of algebraic structures on abstract sets (i.e. sets in
general) is the main concern of Abstract Algebra.
Note. (i) The algebraic structure that a set A receives
depends on the type of the binary operation „o‟ defined over A.
Also the algebraic structure that a binary operation „o‟ imposes
on a set A depends on the elements of A. As a result, the same set
A can be made to have different structures by defining the binary
operation in different ways while the same binary operation „o‟
can impose different structures on different sets. Hence in the
study of algebraic structures, the set A and the binary operation
„o‟ over A have important bearing.
(ii) In the explanation of structures, usually we take the help of
familiar sets, like the set N of natural numbers, the set R of real
numbers, the set Z of integers, the set Q of rational numbers, the
set C of complex numbers or subsets of the above sets. But the
students should keep in mind that similar structures can be
obtained on abstract sets under suitably defined binary
operations.
38
(iii) In verifying structures on a set which is a subset or R (real
numbers) or C (complex numbers) we take the following facts as
true -
The closure laws for addition and
multiplication, associative and commutative
laws for addition as well as multiplication
and the distributive laws for multiplication
over addition hold in R and C.
39
The set of residue classes modulo m is denoted by Z | (m). Thus
Z | (m) = {(0), (1), (2), (3), …., (m – 1)}
Where (0) = {…., –2m, –m, 0, m, 2m, 3m, ….}
(1) = {…., –2m + 1, –m + 1, 1, m + 1, 2m + 1,
3m + 1, ….
(2) = {…., –2m + 2, –m + 2, 2, m + 2, 2m +
2,…} etc.
(p) = {…., –2m + P, –m + p, p, m + p, 2m + p, where 0 p m–
1, p N
Clearly, the general form of the elements of the class (p) of residue classes
modulo m is km + p where k is an integer positive or negative.
Thus the set Z/(5) of residue classes modulo 5 is given by
Z/(5) = {(0), (1), (2), (3), (4)}
Which consists of five classes (subsets ) and
(0) = { ….., – 10, – 5, 0, 5, 10,….}
(1) = { ….., – 9, – 4, 1, 6, 11,….}
(2) = { ….., – 8, – 3, 2, 7, 12,….}
(3) = { ….., – 7, – 2, 3, 8, 13,….}
(4) = { ….., – 6, – 1, 4, 9, 14,….}.
40
The operation of addition for residue classes of modulo m is sometimes written
as +m or to distinguish it from ordinary addition.
For example
If „+‟ is the operation of addition of modulo classes then for the set of residue
classes modulo 7,
(2) + (3) = (2 + 3) = (5)
(4) + (5) = (4 + 5) = (2)
Because 4 + 5 i.e. 9 is more than 7 and when 9 is divided by 7, the remainder is 2.
Thus the rule for adding two residue classes modulo m is the following –
Add the class numbers a and b of the two classes (a), (b), If it (i.e. a+b) is less
than m, the class number of their addition is a+b. But if a+b is equal or greater than
m, divide a+b by m and write the remainder as the class number.
Complete display of the addition (+) table for the set of residue classes modulo
5 is the following –
column for class (4)
Explanation of the Table. To locate (2) + (3) take the class in the place
determined by the intersection of row for class (2) and column for class (3). The class
at the place of intersection is (0). Thus (2) + (3) = (0).
„ ‟ (multiplication). Let (a) and (b) are two elements of Z/(m). A binary
operation „ ‟, called multiplication of residue classes is defined as follows –
(a) (b) = (ab) = (c)
41
Where c is the remainder when ab is divided by m, i.e., ab = km + c where k is any
integer.
The operation of multiplication for residue classes is sometimes written
as or m to distinguish it from oridinary multiplication.
For example
If „ ‟ is the operation of multiplication of modulo classes then for the set of
residue classes modulo 6,
(1) (4) = (1 4) = (4)
(2) (3) = (2 3) = (6) = (0)
(3) (5) = (3 5) = (15) = (3)
Because 2 3, 3 5 i.e. 6, 15 when divided by 6 leave the remainders 0, 3
respectively.
Thus the rule for multiplying two residue classes modulo m is the following –
Multiply the class numbers a and b of the two classes (a), (b). If it (i.e. ab) is
less than m, the class number of their multiplication is ab. But if ab is equal or
greater than m, divided ab by m and write the remainder as the class number.
Complete display of the multiplication ( ) table for the set of residue classes
modulo 6 is the following –
42
Explanation of the Table. To determine (3) (5) take the class in the place
located by the intersection of the row for class (3) and column for class (5). The class
at the of intersection is (3).
Thus (3) (5) = (3).
43
„+‟ and „.‟ Being ordinary addition and multiplication.
Verify which of the algebraic laws does the operation „o‟ satisfy over the set Q
– {1}.
Solution. If a, b are rationals other than 1 then a + b – ab is also rational
other than 1.
a, b Q – {1} a o b Q – {1}.
The closure law holds.
Here a o b = a + b – ab
b o a = b + a – ba = a + b – ab
a o b = b o a.
Hence the commutative law holds.
Next (a o b) o c = (a + b – ab) o c
= (a + b – ab) + c – (a + b – ab) c
= a + b – ab + c – ac – bc + abc,
as associative and distributive laws hold,
= a + b + c – ab – ac + abc,
as associative laws holds for ordinary addition,
and a o (b o c) = a o (b + c – bc)
= a + (b + c – bc) – a (b + c – bc)
= a + b + c – bc – ab – bc + abc,
as associative and distributive laws hold
= a + b + c – ab – ac – bc + abc,
as associative law holds.
Thus (a o b) o c = a o (b o c). Hence the associative law for holds over Q.
The identity element e for the operation „o‟ will exist if
aoe=eoa=a
i.e. a + e – ae = e + a – ea = a.
As commutative laws for addition and multiplication hold we get only
a + e – ae = e
44
or e – ae = 0; or e(1 – a) = 0.
If a a 1, e = 0. If a = 1, e is indeterminate.
Thus the set Q – {1} has the identity element 0.
If possible let b be the inverse element of a.
Then a o b = b o a = e = 0.
a + b – ab = 0; or b(a – 1) = a.
a
b Q {1}.
a 1
a
The inverse element of a exists and it is .
a 1
2. Construct the tables for addition and multiplication of the residue
classes modulo 4. What are the laws that the operation of addition satisfies over the
set Z/(4)? Is multiplication distributive over addition? Verfiy.
Solution. The residue classes modulo 4 are (0), (1), (2), (3)}. So
Z/(4) = {(0), (1), (2), (3)}.
Addition table –
Multiplication table –
45
From the addition table it is clear that the addition of any two members of (0),
(1), (2), (3) is also one among these four. Hence the closure law for addition holds.
Clearly, if (a) Z/(4) and (b) Z/(4)
then (a) + (b) = (a + b) =(b + a),
because commutative law holds. For integers =
(b) + (a).
[Note. This is true even when a + b 4.]
Commutative law for addition holds.
Let (a), (b), (c) Z/(4).
Then {(a) + (b) + (c) = (a + b) + (c) = ((a + b) + c)
= (a + (b + c)),
Because associative law holds for integer
= (a) + (b + c) = (a) + {(b) + (c)}.
Associative law holds for addition.
Observing the row and column for (0) in the addition table we get (a) + (0) =
(0) + (a) = (a) for all (a) Z/(4).
(0) is the identity element of addition. Hence the identity element exists.
Next, from the table we get
(0) + (0) = (0)
(1) + (3) = (3) + (1) = (0)
(2) + (2) = (0).
Thus the inverse elements of (0), (1), (2), (3) exist and they are (0), (3), (2), (1)
respectively.
Again, for (a), (b), (c), Z/(4) we get
(a) {(b) + (c)} = (a (b + c)
= (a (b + c)) = (a b+a c),
{ distributive law holds for integers}
= (a b) + (a c) = (a) (b) + (a) (c).
46
This holds even when b + c 4. For instance,
(2) {(3) + (2)} = (2) (1) = (2)
(2) (3) + (2) (2) = (2) + (0) = (2)
(2) {(3) + (2)} = (2) (3) + (2) (2).
Thus multiplication is distributive over addition.
EXERCISE
1.(i) In a set R of real numbers, show that the operation defined by a b=
a + 2b, for all a, b R is not associative. Is „ ‟ commutative in R?
(ii) On the set Z of integers, a binary composition
„o‟ is defined as a o b = a – b. Prove or disprove the statement that the
binary operation „o‟ is neither associative nor commutative.
(iii) On the set of real numbers R, a binary operation „ ‟
is defined as follows -
na b
a b ,(n 1, 1).
n 1
Is „ ‟ associative or commutative? Verify.
2.(i) Construct the addition and multiplication tables for the residue classes
modulo (7).
(ii) Construct the multiplication table for the set
G = {1, – 1, i, – i} where i2 = – 1.
Does G satisfy closure law?
(iii) Construct the multiplication table for the set of
residue classes modulo 6. What laws do the non-zero residue classes
modulo 6 satisfy for multiplication?
47
GROUP
Introduction
The simplest algebraic structure on a set is the group structure. Thus is a
structure imposed on a set with only one binary operation (composition).
Definition of a Group.
A non-empty set G, together with a binary operation „‟o‟ is called a group (G,
o) if the following laws hold for the operation „o‟-
(G1) Closure law. G is closed under the binary operation
„o‟ i.e. a o b G for all a, b G.
(G2) Associavtie law. The associative law for the operation
„o‟ holds i.e.(a o b) o c = a o (b o c) for all a, b, c G.
(G3) Existence of Indentity Element. The identity element
exists in G i.e. there exists an element e G such that
a o e = e o a = a for all a G.
(G4) Existence of Inverse Element. The inverse element of
all the elements of G exists i.e. for every a G there exists an element
b G such that a o b = e o a = e, e being the identity element.
The identity element of a is denoted by a-1 and so
a o a-1 = a-1 o a = e.
Note. The law () also assures (a o b) o (c o d)
= a o (b o c) o d
= {a o (b o c)} o d
= a o {(b o c) o d}.
For example
Illustration 2.1 The set R of real numbers is a group under the binary
operation „+‟ (ordinary addition) i.e. (R, +) is a group.
Solution. (i) Clousre law holds because sum of any two real
numbers is a real number i.e. a + b R if a, b R.
48
(ii) Associative law holds because we know
a + (b + c) = (a + b) + c
Where a, b, c, are any three real numbers.
(iii) Indentity element 0 exists because 0 R and
a + 0 = + a = a for every real number a.
(iv) Inverse element exists because for every
a R there is – a R such that a + (– a) =
(– a) + a = 0.
Thus (R, +) is a group.
Illustration 2.2 The set R of real numbers is a group under the binary
operation „ ‟ (ordinary multiplication) i.e. (R, ) is a group.
49
the following multiplication table –
1 –1 i –i
1 1 –1 –i –i
–1 –1 1 –i i
i i –i –1 1
–i –i i 1 –1
50
(iv) In illustration 2.3, (G, ) is a group but (G, +) is
not, though the underlying set G is the same. Thus on the same set,
a binary operation can impose group structure while another binary
operation fails to do so.
Thus considering the notes (ii), (iii) and (iv) we can conclude the following –
The group structure is not inherent in any set and no binary operation has the
capacity to impose group structure on every set. It depends on the elements that make
the set and the binary operation defined on the set. As such the pair (G, o) of the set
and the binary composition is used to indicate a group.
(v) In the definition of a group we can dispense with
(G1) i.e. closure law if we describe the binary operation „o‟ to be
denned over G. To say „o‟ is defined over G is to say closure law
holds for ‘o’.
not, though the underlying set G is the same. Thus on the same set,
a binary operation can impose group structure while another binary
operation fails to do so.
(vi) Description of a group in terms of more primitive structures-
Groupoid or Quasi-group. A non-empty set G, together with a binary operation
‘o’ is called a groupoid (G, o) if G is closed under ‘o’ i.e. (G1) of § 2.1 holds.
Semi-group. A groupoid (G, o) is called a semi-group if the associative law
holds. Thus a semi-group satisfies (G1) and (G2) of § 2.1.
Monoid. A semi-group (G, o) is called a monoid if identity element exists.
Thus a monoid satisfies (G1), (G2) and (G2) of § 2.1.
Group. A monoid (G, o) is called a group if inverse element for each element
of G exists. Thus a group satisfies (G1), (G2), (G3), and (G4) of § 2.1.
It is clear that every group is a monoid, a semi-group and a groupoid. Every
moniod is a semi-group and a groupoid but may not be a group. Every semi-group is a
groupoid but may not be a monoid or a group.
51
Abelian Group or Commutative Group
A group (G, o) is called an abelian group or commutative group if the
commutative law holds for the operation.
In more clear terms, a non-empty set G, together with a binary operation „o‟ is
called an abelian group (G, o) if (G1), (G2), (G3), (G4) of § 2.1 hold and the following
law holds for the operation „o‟–
(G3) Commutative law. The commutative law for the operation „o‟ holds i.e. a
o b = b o a for all a, b G.
Note. Every abelian group is a group. But all groups are not
abelian.
The name „abelian‟ is derived from N.H. Abel (1802–1829), a Norwegian
mathematician.
For example
Illustratioin 2.4 The set T of all non-zero rational numbers is an abelian group
under (usual multiplication).
p
Solution. Let a, b, c T. As rational numbers are of the form
q
p1 p2 p3
We take a ,b ,c where P1, P2, P3, q1, q2, q3, are non-zero integers.
q1 q2 q3
p1 p2 p1 p2 p
Now a. b . is of the form .
q1 q2 q1q2 q
p1 p2 p3
, for associative law holds for the set of integers.
q1q2 q3
p1 p2 p3 p1 p2 p3
. . . (a. b).c.
q1q2 q3 q1 q2 q3
52
p p
1 T and a 1 1 1 1 a.
q q
1 1
p p
1 a 1 1 1 a.
q q
1 1
So a 1=1 a = a.
Hence 1 is the identity element.
p p
For every a 1 there exists 1 T such that.
q q
1 1
p p
1 is 1 .
q q
1 1
p p
The inverse element of 1 is 1 . Thus inverse elements exist.
q q
1 1
p1 p2 p1 p2
Lastly a. b. .
q1 q2 q1q2
p2 p1 p2 p1 p1 p2
b. a) . ,
q2 q1 q2 q1 q1q2
53
i.e. A (BC) = (AB) C
when A (BC) and (AB) C are defined.
As A, B, C are all n n matrices, A (BC) and (AB) C
are defined and each is a n n non-singular matrix.
(iii) The identity element is the unit n n matrix I, for we
know AI = IA = A (by property of unit matrix) and I is non-singular as
|I|=1 0.
(iv) We know for non-singular matrices, corresponding to each n n non-
singular matrix A there exists a n n non-singular matrix A–1 (reciprocal
matrix of A) such that
AA–1 = A–1 A = I.
So the inverse elements exist.
M is group under matrix multiplication.
But we know from matrix multiplication, AB BA in general.
So commutative law does not hold.
Thus M is a non-abelian group under matrix multiplication.
Properties of a Group
On the basis of the laws that hold on a group (G, o), group structure depicts
many properties similar to those holding in the real number set R.
Remember in many places we shall write a o b as ab only for convenience.
Property 1. (Uniqueness of the Idnetity Element)
To prove that the identity element of a group is unique.
Proof. Let (G, o) be a group, with the identity element e G, We have to prove
that identity element is unique.
If possible let e’ G be another identity element of the group.
As e is the identity element, by definition
a o e = e o a = a for al a G.
54
As e’ G, putting e’ for a in the above, we get
e’ o e = e’ o a = a. …(1)
By assumption e’ is also an identity element of G.
By definition, a o e’ = e’ o a = a.
As e G, e o e’ = e’ o e = e. …(2)
By (1) and (2) we get e = e’.
Hence any other identity element of G is not different from e. So e is the unique
identity element.
Property 2. (Uniqeness of Inverse Element)
To prove that the inverse element of an element in a group is unique.
Proof. Let (G, o) be a group and a G,
We have to prove that the inverse element a–1 G of a is unique. If possible let
a’ G be another inverse element of a.
As a–1 is the inverse element of a, by definition.
a o a–1 = a–1 o a = e,
e being the identity element of the group.
a’ o (a o a–1) = a‟ o e …(1)
{using only a a o a–1 = e}.
But a‟ o e = e‟, for e is the identity element and a’ G.
Also a’ o (a o a–1) = (a’ o a) o a–1, for associative law holds in a group
= e o a–1, { a’ is also an inverse of a }
= a–1.
Putting these in (1), a–1= a‟ i.e. a’ is not different from a–1. Hence the
inverse element a–1of a is unique.
Note. (i) Inverse element of e, the identity element, is e itself for
e o e = e.
55
i.e. (a–1)–1 a for a–1 o a = a o a–1 = e
and consequently by definition of inverse
element a is the inverse of a–1.
56
(–i)4 = 1; so the order of i is 4, i.e. O(–i) = 4.
(1)1 = 1; so the order of 1 is 1, i.e. O(1) = 1.
(–i)2 = 1; so the order of –1 is 2, i.e. O(–1) = 2.
Illustration 2.9. The set Z of integers is a group under usual addition. For
integer a 1 (i.e. the indentity element) there is no positive integer n such that
a + a + a + ….. up to n terms = 1.
Order of a ( 1) is zero (or infinite).
Cyclic Group
Definition of Cyclic group. A group (G, o) is called a cyclic group if there
exists an element such that every element b of G can be expressed as an integral.
(positive or negative) power an of a. The element a whose powers exhaust the set G is
called a generator of the cyclic group.
Thus a cyclic group consists only of integral powers of a particular element of
the group.
For example
3 3
Illustration 2.10. The set G = {1, , } where = 1, = 1 i.e. the set of
cube roots of unity is a cyclic group under multiplication of complex numbers.
By property of cube roots of unity if = w then = w2. Also w2 = 1.
Obviously G is a group under multiplication of complex numbers, the identity
element being 1 and
( )–1 = w2 = , ( )–1 = w = a.
Clearly G = {w3, w, w2}. So each element of G is an integral power of w G.
Hence G is cyclic group under multiplication of complex numbers.
57
Definition of Subgroup.
If G be group under the operation ‘o’ and H is a non-empty subset of G and H
is a group itself under the same operation ‘o’ then the group (H, o) is called a
subgroup of the group (G, o).
Thus the group (H, o) is a subgroup of the group (G, o) if H G and H is non-
empty.
Illustration 2.13. The set E of even integers is a subgroup of the group Z of
integers under ordinary addition.
Solution. Clearly the Z of integers is a group under ordinary addition, 0 being
the identity element and – a Z being the inverse element of a Z.
Now b E means b = 2a where a Z. Clearly F Z.
Also it can easily be verified that E is a group under ordinary addition, 0 being
the identity element and – b E being the inverse element of b E.
Thus E Z and E is a group under the same operation as that on Z.
Hence (E, +) is a subgroup of the group (Z, +).
Illustration 2.14. The set R+ of positive real numbers is a subgroup of the
group R of real numbers under ordinary multiplication.
Solution. Clearly the set R of real numbers is a group under multiplication,
1 R being the identity element and is the inverse element of a R.
58
(ii) If H is a non-empty subset of G where (G, o) is a group then H is
called a complex of the group. Thus if (H, o) is a subgroup of the group
(G, o), H is a complex of the group.
Ring
Definition of a Ring
A non empty set R with two binary operation, „+‟ (called addition) and ‘o’
(called multiplication) is said to be a ring (R, +,.) if (R, +) is an abelian group, (R,.) is
a semi-group and „‟ is distributive over „+‟.
In more explicit terms the definition is as follows–
For + (addition)
(P1) Closure law. a, b R (a + b) + c = a + (b + c).
(P2) Associative law. a, b, c R (a + b) + c = a +(b+c).
(P3) Commutative law. a, b R a + b = (b + a).
(P4) Existence of Identity Element. There exists an element 0 R (called
zero element) such that
a + 0 = a for all a R.
(P5) Existence of Inverse Element. Corresponding to each element a R
there exists an element x R, called additive inverse or negative
element, such that
a + x = 0.
(P6) Closure law. a, b R a. b R.
(P7) Associative law. a, b, c R (a . b) . c = a (b. c).
59
For + and.
(P8) Distributive law. a, b, c R a.(b+ c) = a. b + (a .
c and (b + c). a = b . a = b. a + c . a.
Note. (i) The postulate connects the two operations „+‟
and „.‟.
(ii) As the commutative law for addition holds in a
ring, the P0 stulate (P4) can be written as a + 0 = 0 + a = a for all
b R and postulate (P5) can be written as
a+x=x+a=0
(iii) In this chapter the letter R is used to denote the
set of the ring.
(iv) The operation under which R is an abelian group is termed as the
operation of addition.
For example
Illustration 2.15. The set Z of integers is a ring under ordinary addition and
multiplication.
Solution. As the addition of two integers is an integer, the closure law holds for
„+‟. Also we know that the associative and commutative laws of addition hold for
integers.
The zero element in Z is 0. The additive inverse of a Z is – a Z. Hence
(Z,+) is an abelian group.
Again the multiplication of two integers is an integer. So the closure law holds
for „.‟. Also we know that the associative law holds for multiplication in Z.
Again we know for any three integers a, b, c we have
a . (b + c) = a . b + a . c
and (b + c) . a = b . a + c . a.
Hence (R, +,.) is a ring.
60
Illustration 2.16. The set E = {x | x = 2m, m Z} is a ring under ordinary
addition and multiplication.
Solution. Let a, b E. Then c = 2m1, b = 2m2, where m1,
m2 Z. Clearly a + b = 2m1 + 2m2 = 2 (m1 + m2) E.
Closure law holds for addition.
Again a . b = 2m1 . 2m2 = 2 (2m1 . m2) E.
Closure law holds for multiplication.
As the E Z and the commutative law, the associative law hold for addition in
Z and associative law for multiplication holds for multiplication in Z, these laws holds
for E also.
The zero element is 0.
The additive inverse of a E. is – a E.
Also the distributive law holds in Z and E Z.
Thus (E, +,.) is a ring.
Some Definition
(i) Commutative ring. A ring (R, +,.) is called a commutative ring if the
commutative law for multiplication holds in R i.e. (P9) a, a R a.b
= b. a.
In explicit terms, a non-empty set R is called a ring under the operating,
„+‟ (addition) and „.‟ (multiplication) if the postulates (P1), (P2), …..
(P3), hold in R.
A ring which is not commutative is called a non-commutative ring.
(ii) Ring with Unity. A ring (R, +,.) is called a ring with unity if there exists
an element e R such that
a . e = e . a = a for all a R
i.e. identity element for multiplication exists in R.
If no such element exists in R then we call it a ring without unity.
61
For example
Illustration 2.17 (Example of a Commutative ring with Unity)
The set R of real numbers is a commutative ring with unity under ordinary
addition and multiplication.
Solution. As the addition and multiplication of two real numbers is a real
number, the closure law holds for addition as well as multiplication.
Also the associative law and commutative law for addition as well as
multiplication hold for real numbers.
The zero element in R is 0.
The additive inverse of a R is – a R.
Also for any three real numbers a, b, c we have
a (b + c) = ab + ac
and (b + c) a = ba + ca.
The distributive law holds in R.
Thus R is a commutative ring.
Again the identity element of multiplication is 1 R.
R is a commutative ring with unity with respect to ordinary addition and
multiplication.
Illustration 2.18 (Example of a Commutative ring without unity)
The set of even integers (zero, positive and negative) is a
commutative ring without unity under ordinary addition and multiplication.
Solution. See illustration 2.16.
Thus the set E of integers is a ring under ordinary addition
and multiplication.
Also a . b = 2m1 . 2m2 = 4m1m2
and b . a = 2m2 . 2m1 = 4m2m1 = 4m1m2.
a . b = b . a i.e. commutative law for multiplication holds in E. So (E,
+,.) is a commutative ring.
But there is no element e in E such that
62
a.e=e.a=a
for e a = a e=1 E.
(E, +,.) is a commutative ring without unity.
Integral Domain
Definition of Integral Domain
A commutative ring with unity of at lest two elements is called an integral
domain if there is no divisors of zero in the ring.
Thus an integral domain I is a ring under two binary compositions addition and
multiplication if the following hold –
(i) I is a commutative ring.
(ii) I has unity (i.e. the identity element of multiplication)
(iii) I has no divisors of zero.
More explicitly, a set I (with at least two elements) is called an integral domain
under the binary operations, addition (+) and multiplication (.) if the following
postulates hold –
For addition (+)
(P1) Closure Law. a, b I a+b I.
(P2) Commutative Law. a + b = b + a for all a, b I.
(P3) Associative Law. (a+ b) + c = a+(b+ c) for all a ,b,
c I.
63
For multiplication (.)
(P6) Closure law. a, b I a. b I.
(P7) Associative law. a . (a . b) = (a . b) c for all = a, b
c I.
(P8) Commutative Law. a . b = b . a for all a, b I.
(P9) Existence Law of Identity. There exists an element
1 I (called unity) such that
a . 1 = a for all a I.
(P10) Absence of Divisors. There exists an element a. b = 0
either a = 0 or b = 0.
For addition (+) and multiplication (.)
(P11) a. (b + c) = a . b + a . c,
(b + c) . a = b. a + c . for all a, b, c I.
Note. Clearly every Integral Domain is a ring. But a ring is
not necessarily an integral domain.
For example
Illustration 2.20 The set Z of integers is an integral domain under ordinary
addition and multiplication.
Solution. As in illustration 3.1, it is a ring.
Now, for two integers a, b we know
ab = ba and ab = 0 either a = 0 or b = 0.
The commutative law holds and divisors of zero are absent. Also 1 Z
is the unity element.
Z is an integral domain under ordinary addition and multiplication.
Illustration 2.21 The set Z/(p) of residue classes modulo p is an integral
domain under addition and multiplication of residue classes if p is a prime number but
it is not an integral domain if p is a composite number.
The zero element = (0), unity element = (1).
64
Additive inverse of (m) = (p – m).
Let n 0. We have to prove that n is a prime number.
If not, let n=n1n2 where 1<n1 <n, 1<n2 <n.
Now (n1n2) a= 0 for all a I.
As the unity element e I, we get
(n1n2) e = 0 i.e. (n1n2) e. e=0, { e . e = e}
(n1e) (n2e) = 0, { by distributive law,
( e + e + .… to n1 times) . ( e + e + … to n2 times)
= ( e . e + e . e + ..… to n1n2 times}
n1e= 0 or n1e= 0, { divisor of zero is absent in an
integral domain}.
Thus our assumption that n is the least positive integer such that na = 0.
Thus our assumption that n is not prime was wrong.
n is zero or a prime.
FIELD
Definition of Field
A commutative ring F (of at least two elements) with unity is said to be a field
if every non-zero element of F has a multiplicative inverse in F.
Thus field F is a ring under two binary compositions, addition and
multiplication if the following hold –
(i) F is a commutative ring,
(ii) F has unity,
(iii) Every element a ( 0) of F has its multiplicative
inverse in F.
More explicitly, a set F (with at least two elements) is called a field under the
binary operations, addition (+) and multiplication (.) if the following postulates hold –
65
For addition (+)
(P1) Closure Law. a, b F a+b F.
(P2) Commutative Law. a + b = b + a for all a, b F.
(P3) Associative Law. (a+ b) + c = a+(b+ c) for all a ,b,
c F.
(P4) Existence Law of Identity. There exists an element
0 F (called zero element) such that a + 0 = a for all a F.
(P5) Existence Law for Inverse Element. a F implies there exists an
element x I (called additive inverse or negative element) such that
a + x = 0.
The additive inverse of a written as – a.
For multiplication (.)
(P6) Closure law. a, b F a. b F.
(P7) Associative law. a . (a . b) = (a . b) c for all = a, b
c F.
(P8) Commutative Law. a . b = b . a for all a, b F.
(P9) Existence Law of Identity. There exists an element
1 F (called unity) such that
a . 1 = a for all a F.
(P10) Existence Law of Inverse for Non-zero Elements.
a( 0) F implies there exists an element x F (called multiplicative
inverse or reciprocal element) corresponding to a such that
a . x = 1.
1
The multiplication inverse of a is written as a 1 or .
a
For addition (+) and multiplication (.)
(P11) Distributive Law. a. (b + c) = a . b + a . c,
(b + c) . a = b. a + c . for all a, b, c F.
Note. Every field is necessarily a ring. But a ring may not
be a field.
66
For example
Illustration 2.22 The set R of real numbers is a field with respect to ordinary
addition and multiplication.
Solution. By illustration 3.3, the set R of real numbers is a commutative ring
with unity with respect to ordinary addition and multiplication.
1
Also if a ( 0) R then the real number is the multiplicative inverse of a. So
a
every non-zero element of R has its multiplicative inverse in R.
Thus R is field with respect to ordinary addition and multiplication.
(group)
(set) (binary operation)
(set) *
(closure) - ∀ a, b ∈ G ⇒ a * b ∈ G
(commutative) - ∀ a, b, c ∈ G ⇒ a * (b * c) = (a * b) * c
(existence of identity) –
∃ e ∈ G, s.t. ∀ a ∈ G => a * e = a = e * a
67
(existence of inverse) –
a∈G b ∈ G s.t. a * b = b * a = e
(group) (G, *)
b R a. b R (Associative) a, b, C G a (b
c) = (a b) c
a. (b + c) = a.b + a.c
(a + b).c = a.c + b.c Field
(F, + ,.)
68
CHAPTER - 3
MATRICES
Objectives
Unit
Basic operation of matrix algebra
Transpose of matrix
Singular and non-singular matrices
Symmetric and skew symmetric matrices
Conjugate of matrix
Hermitian and Skew-hermition matrices
Adjoing of matrix
Inverse of matrix
Orthogonal matrix
Rank of the matrix
INTRODUCTION
The subject of matrices had its origin in various types of linear problems, the
most important of which concerns the nature of solution of any given system of linear
equations and linear transformations in geometry. In the beginning it was thought as it
the subject of matrices was only a development of that of determinants, with the
passage of time, however the subject of matrices has found application to very large
number of disciplines such as Geometric, Statistics, Chemistry, Physics, Psychology,
Education etc.
69
Basic operations of Matrix Algebra :-
Students are familiar with the different types of matrices and basic operations
during intermediate class. Some of them require more attentions which include the
laws follows by these operations such as associative law commutative.
(i) Addition of matrices or Sum of matrices
The sum of two matrices of the same order A = [aij] and B = [bij] is defined to
be the matrix
C = [cij] such that cij = aij + bij and it is denoted by A + B.
Thus the sum of two m n matrices as m n matrix found adding
corresponding elements clearly, addition of two matrices is defined only for
two matrices of the same order. Two matrices which can be added are called
conformable for addition. In a simple language, the matrices A and B are
conformable for addition only when both have the same number of rows and
same number of column.
2 3 7 1 3 2
Example :- Let A= B=
4 6 5 2 0 4
Here, the number of rows of A = 2 = the number of rows of
B and the number of column of A = 3 = the number of column of B.
Hence A and B are conformable for addition.
2 3 7 1 3 2
A+B= +
4 6 5 2 0 4
2 1 3 3 7 2
=
4 2 6 0 5 4
3 6 9
=
6 6 9
70
Proof:-
Since the matrices A, B and C are conformal for addition, therefore they must
be of the same order.
= A + (B + C).
Hence (A + B) + C = A + (B + C)
Thus the associative law for addition of matrices hold.
(iii) Commutative law of matrix addition.
If the matrices A and B are conformal for addition, then
A+B=B+A
Proof:- Since matrices A and B are conformal for addition,
therefore they must be of the same order. Hence the sum A + B and B + A are
defined. Let
A aij and B bij
Be of the order m n.
71
Now A + B = aij bij bij aij bij aij
=B+A
Hence A+B=B+A
Thus the commutative law for addition of matrices hold.
(iv) Multiplication of a matrix by a scalar
Let A be a matrix. The matrix obtained by multiplying every element of the
matrix A by a scalar k is called the product of A by k and is denoted by kA or
Ak.
1 2 3 5 10 15
Example:- If A then 5A
4 5 6 20 25 30
(v) Negative of a matrix:- The negative of a matrix A =[a ij] is defined as the
matrix B such that A + B = 0 (null matrix)
Thus B = [–aij] ie (–1) [–aij] = (–1) A
The negative of a matrix A is defined by – A
2 3 4 2 3 4
Example:- If A Then A
3 5 7 3 2 7
72
4 6 5 2 3 1
Then A B
2 3 4 3 2 2
4 2 6 3 5 1 2 3 4
2 3 3 2 4 2 1 1 2
= kaij kbij kA kB
Hence k (A+B) = kA + kB
Similarly, (A+B) k = Ak + Bk.
(viii) Definition for matrix multiplication
Two matrices A and B are conformable for product AB if the number of
columns of A is equal to the number of rows of B. The (ij)th element of
product AB is the sum of the products of the element of ith row of A with
corresponding elements of jth column of B.
Thus If A = [aij] be a m n matrix.
and B = [bij] be a m p matrix.
Then product AB is possible and
n
AB aij bik will be m p matrix.
j 1
73
If the product AB is denoted by the matrix C=[cik]
Then AB = [cik] is an m p matrix.
Where Cik = (ik)th element of AB
n
= aij bik
j 1
a1 b1 x1 x2 x3
But a2 b2 y1 y2 y3
a3 b3 z3 z2 z3
has no meaning because the number of columns of first matrix is 2 which not
equal to the number of rows of the second matrix equal to 3.
(ix) Non commutative law of matrix multiplication.
The multiplication of matrix is not necessarily commutative.
It is sufficient to prove by an example that A B B A.
74
1 0
1 0 0
Let A 0 1 and B
0 1 0
0 0
Since A and B are square matrices of the same order therefore, the product AB
can be obtained.
1 1 1 0 0 0
AB 0
1 1 1 1 0 0
But neither A nor B is equal to zero. Thus the product of two non-zero matrices
can be a zero matrix.
(xi) Idempotent matrix and Nilpotent matrix
A square matrix A is called idempotent if A2 = A.
If AP= 0 (null matrix), where p is a positive integer then this matrix is called
nilpotent. If however, p is the least integer for which AP= 0 (null matrix), then
p is the index of the nilpotent matrix A
(xii) Associative law for multiplication
75
If the matrices A and B are conformal for the product AB and the matrices B
and C are conformal for the product BC the (AB) C = A (BC).
Proof:-
Let A = [aij], B =[bjk] and C = [ckl] be the matrices of order m n, n p and
p q respectively.
Hence A and B are conformal for the product AB
AB = [aij] [bij]
p
= u jl bik ckl …………………………..(1)
k 1
n
Where ik aij bik is the (i, k)th element of AB
j 1
p n
= aij bik ckl
k 1 j 1
p n
= aij bik ckl ………………..(2)
k 1 j 1
Which is an m q matrix
Next, B = [bik] and C = [ckl] are n p and p q matrices repectively.
Therefore, B and C are conformal for the product BC.
BC = [bik] [ckl]
76
p
= bij ckl u jl ………………..(3)
k 1
p
Where u jl bik ckl = is the (il)th element of BC.
k 1
But A is an m n matrix. Hence A and (BC) are conformal for the product
A(BC).
n
BC = [aij] [ujl] = aij u jl
j 1
n p n p
= aij b jk ckl aij bik ckl
j 1 k 1 j 1k 1
n p
= aij bik ckl ………………..(4)
j 1k 1
Which is an m q matrix
From (2) & (4) conclude that (AB)C = A (BC)
(xiii) Distributive law for matrix multiplication:-
To prove that A (B+C) = AB where A, B and C are conformal for the
operations involved.
Proof:-
Let A = [aij], B =[bjk], C = [ckl]
be the matrices of order m n, n p and n p respectively.
Since B and C are conformal for addition, therefore
B + C = [bjk] + [cjk] = [bjk + cjk]
Which is an n p matrix. Clearly A and B+C are conformal for the product A
(B+C).
A (B+C) = aij b jk c jk
77
n
= aij b jk c jk
j 1
n n
= b jk ckl aij c jk ………………..(1)
j 1 j 1
n n
AB+AC = aij b jk aij c jk ………………..(2)
j 1 j 1
Workedout Examples:-
2 4 3 3 5 6
1. if A 0 1 5 , B 2 3 2
3 2 4 1 4 3
Find A+B and B+A and verify the commutative law of additions for them
matrices.
78
Solution:-
Since A and B are square matrices of the same order therefore, A+B and B+A
both exist.
2 4 3 3 5 6 1 9 9
A B 0 1 5 2 3 2 2 4 3
3 2 4 1 4 3 4 6 7
3 5 6 2 4 3 1 9 9
B A 2 3 2 0 1 5 2 4 3
1 4 3 3 2 4 4 6 7
3 2 4 4 7 6 5 8 13
5 5 4 1 6 3 10 5 9
3 1 1 6 2 7 2 7 5
5 8 13 2 0 1
( A B) C 10 5 9 3 2 7
2 7 5 0 2 1
79
5 2 8 0 13 1 3 8 14
10 3 5 2 9 7 13 3 16 …………..(1)
2 0 7 2 5 1 2 9 4
2 4 6 2 0 1
Again B+C 5 1 3 3 2 7
1 6 7 0 2 1
2 2 4 0 6 1 0 4 7
5 3 1 2 3 7 8 1 10
1 0 6 2 7 1 1 8 6
3 4 7 0 4 7
A (B C) 5 4 6 8 1 10
3 1 2 1 8 6
3 8 14
13 3 16 ………………..(2)
2 9 4
1 3 4 2 4 6 3 4 5
Solution:- 2 3 3
2 4 5 1 2 3 2 3 4
2 6 8 6 12 18 9 12 15
4 8 10 3 6 9 6 9 12
2 6 9 6 12 12 8 18 15
4 3 6 8 6 9 10 9 12
5 6 11
7 11 13
80
4. Find the product AB and BA, and show that AB BA
1 1 1 1 2 3
A 2 2 1 and B 2 4 6
3 1 0 1 6 3
Solution:-
Since A and B are square matrices of the same order, therefore, products AB
and BA are possible.
1 1 1 1 2 3
AB 2 2 1 2 4 6
3 1 0 1 6 3
0 4 0
1 2 3
1 2 3
1 2 3 1 1 1
And BA = 2 4 6 2 2 1
1 6 3 3 1 0
12 6 1
24 12 2
20 14 5
1 2 3 x
4. If A ,B ,C
3 4 4 y
81
Solution:- obviously AB and AC are defined and
1 2 3 1.3 2.4 11
AB
3 4 4 3.3 4.4 25
and AC
Since AB = AC which implies that
11 x 2y
25 3x 4 y
x + 2y = 11 ………………………..(1)
3x + 4y = 25 ………………………..(2)
Multiplying (1) by 2 and subtracting from (2), we get
3x + 4y = 25
2x + 4y = 22
x=3
2y = 11–3 = 8 or y = 4
Hence x = 3, y = 4
2 1
6. If A find the value of A2 – 4A + 3I, where I is the unit matrix with
1 2
proper dimension.
2 1
Solution:– Here A
1 2
2 1 2 1
A2 = A.A
1 2 1 2
4 1 2 2 5 4
2 2 1 4 4 5
82
5 4 2 1 1 0
= 4 3
4 5 1 2 0 1
5 4 8 4 3 0
=
4 5 4 8 0 3
5 8 3 4 4 0 0 0
= 0
4 4 0 5 8 3 0 0
Hence A2 –4A + 31 = 0
1 1 1 1 3
1 2 3 4
7. If A 2 0 3 , B 0 4 and C
2 0 2 1
3 1 2 1 4
2 1
1 2 3 4
(AB) C 1 18
2 0 2 1
5 3
83
4 4 4 7
35 2 39 22 …………………….(1)
11 10 9 17
7 2 3 1
4 0 4 2
7 2 11 8
1.7 1.4 1.7 1.2 1.0 1( 2) 1.( 3) 1.( 4) 1( 11) 1.( 1) 1.2 1.8
2.7 0.4 3.7 2.2 0.0 3.( 2) 2.( 3) 0.( 4) 3( 11) 2( 1) 0.2 3.8
3.7 1.(4) 2.7 3.2 1.0 2.( 2) 3.2( 3) 1.( 4) 2( 11) 3.( 1) 1.2 2.8
4 4 4 7
4 2 39 22
11 10 9 17
84
Transpose of a matrix:-
(i) Definition:-
Let A be a given matrix of the type m n then the matrix obtained by changing
the rows of A into columns or columns of A into rows is called transpose of
matrix A and is denoted by A‟ or AT.
And the matrix A‟ is of type n m.
3 4
3 2 5
For example:- (i) If A 2 1 then A '
4 1 9 2 x3
5 9 3x2
85
Proof:- Let A = [aij] and B = [bij] be two matrices conformal
for addition. Then we have to prove that
(A+B)‟ = A‟+B‟
A = [aij] therefore by the definition of transpose of a matrix.
A‟ = [aji]
Again B = [bij]
B‟ = [bji]
From A +B = [aij] + [bij]
= [aij + bij] = [cij] (say)
(A+B) = [cij]‟ = [cji]
= [aij + bij] = [aij] + [bji] = A‟ + B‟
Hence (A+B)‟ = A‟ + B‟
3. If A is matrix and k be any non zero scalar, it is to prove
that (kA)‟ = kA‟
Proof:- Let A = [aij] then A‟ = [aji]
Now kA = k[aij] = [kaij]
(kA)‟ = [kaij]‟ = [kaji] = k [aji] = kA‟
Hence (kA)‟ = kA‟
4. Reversal rule for transpose.
To prove that the transpose of the product of two matrices is equal to the
product of their transposes in the reverse order.
i.e. (AB)‟ = B‟A‟, where matrices A,B are conformal.
Proof:- Let A = [aij] and B = [bjk] be the matrices of the order m n and
n p respectively.
86
n
aij b jk which is an m p matrix.
j 1
=[cik]
n
So, (i k)th element of AB = cik aij b jk
j 1
(AB)‟ is a p m matrix
Again B‟ is p n matrix and A‟ is n m matrix.
So the product B‟A‟ is possible and is of type p m.
Now, (k,i)th element of B‟A‟ is the product of kth row of B‟
and ith column of A‟
kth row of B‟ = kth column of B‟ = (b1k, b2k ……………. bnk).
ith column of A‟ = ith row of A = (ail, ai2 ………………. ain).
Hence (k,i)th element of B‟A‟= blk ail + b2k ai2+ …..+bnk ain
n n
b jk aij aij b jk ……………(2)
j 1 j 1
87
1 2 1
2 3
Example:- If A 2 3 9 ,B
1 4
3 1 10
1 2 1
| A| 2 3 9 1.(30 9) 2(20 27) 1(2 9)
Then
3 1 10
21 14 7 0
88
i.e. aij = aji then it is to prove that A‟ = A
Now, (ij)th element of A‟ = (ji)th element of A
= (ij)th element of A.
A‟ = A
A square matrix B is called skew-symmetric if A‟ = –A. The
Matrix
0
B 0 , Hence – B‟ = B.
0
89
1
A ' A as ( A ') ' A
2
1
A A' X.
2
So X is symmetric
1
Again Y' A A' '
2
1
A ' ( A ') '
2
1
A ' A as ( A ') ' A
2
1
A A' Y.
2
So Y is skew symmetric matrix.
1 1
But X Y A A' A A'
2 2
1
A A' A A'
2
1
2A A
2
So, A = X+Y.
Hence any square matrix A can be expressed as sum of a
symmetric matrix X and skew-symmetric matrix Y.
Next, it is to prove that this representation is unique. Let
us suppose that this representation is not unique.
Let A = X1 + Y1, where X1 ; Y1, are respectively
symmetric and skew-symmetric matrix and X1 X, Y1 Y.
X = (X1 + Y1)‟1 = X1‟ + Y1‟
But X1 is symmetric and Y1 is skew-symmetric.
So X1 X, Y1 =Y.
A‟ = X1 – Y1
90
A‟ + A‟= X1 + Y1 + X1 – Y1 = 2X1
1
X1 A A'
2
Again A‟ – A‟=( X1 + Y1 ) –( X1 – Y1 )
= X1 + Y1 – X1 + Y1 = 2Y1
1
Y1 A A'
2
So we find that X1 = X and Y1 = Y. Which are
contradictory to our assumption.
Therefore our assumption is wrong. Hence the
representation A=X+Y is unique.
(d) If A be a square matrix then to prove that A+A‟ is
symmetric and A–A‟ is skew-symmetric.
Proof:- ( A + A‟ )‟ = A‟ + (A‟)‟ = A‟ + A = A + A‟
Hence A + A1 G symmetric
Again ( A – A‟ )‟ = A‟ – (A‟)‟ = A‟ – A = (A – A‟)
So A–A‟ is skew-symmetric
(e) Prove that the matrix B1AB is symmetric or skew-symmetric as A is
symmetric or skew-symmetric.
Proof:- Case– 1 Let A be a symmetric matrix.
Then A‟ = A
It is to prove that B‟AB is symmetric.
Now, by the reversal law for the transpose, we have
(B‟AB)‟ = B‟(B‟A)‟ = B‟(A‟(B‟)‟) = B‟A‟B
= B‟AB
Hence B‟AB is skew-symmetric
Case-11 Let a be skew-symmetric. So A‟ =-A
Then it is to prove that B‟AB is skew-symmetric
Now (B‟AB)‟ = B‟[(B‟A)‟] = B‟[(A‟B)‟] = B‟A‟B
91
= B‟ (–A)B = –B‟AB
Hence B‟AB is skew-symmetric
Conjugate of Matrix:-
Conjugate of a matrix:- The matrix obtained from any given matrix A by
replacing each of its elements by the corresponding conjugate complex, is
called the conjugate of A. The conjugate of A denoted by A.
Example.
2 2 5i
If A =
3 4i 2 3i
2 2 5i
Then A. =
3 4i 2 3i
92
Hence A = aij is a Hermitian matrix if and only if aij a ji for all i and j
a11 = a11
Hence the matrix given in (i) is Hermitian similarly, it can be shown that the
matrix given in (ii) is also Hermitian
Now consider the matrix in (i) which is Hermitian
4 2 5i
Then A
2 5i 6
4 2 5i
Now transpose of A we get A ' A
2 5i 6
Then A*=A
Skew-Hermitian Matrix
Hence A=[aij] is skew-hermitian if and only aij a ji for all i and j
aij a ji 0
93
aii aii 0 for all i.
iA * Sincei i, A ' A *
(iA)
iA * (iA)
i( A) iA
iA * iA
A' B' A * B * A B.
A B * A B.
94
Proof:- Let A and B be two skew-Hermitian matrices of the same
order.
Then by definition of skew-Hermitian matrix.
We have
A* Aand B * B
Now, we have to prove that A+B is also skew-Hermitian
Hence every diagonal elements of a skew-Hermitian matrix is either zero or a
purely imaginary number
Examples of skew-hermitian Matrix
ia x iy c id
0 2 i
(i) (ii) x iy iy m ip
2 i 0
c id im ip iz
Explanation:
a11= o = o + io
a11 o io a11 o io io
a21 2 i a 21 2 i
0 2 i
Let us consider A which is skew-Hermitian.
2 i 0
0 2 i
A ' A
2 i 0
95
Hence for a skew-Hermitian matrix
A* A
A' B'
A ' B ' A* B* ( A) ( B)
A B
*
A B A B
A A ' '
A' A as A A
A A*
96
*
A A* A A*
A A ' '
A A' as A A
*
AA* AA*
A ' A A* A
A' A ''
97
A' A ' A' A
A* A A* A A A*
*
A A* A A*
98
A A* P1 Q1 P1 Q1 2P1.
A A*
P1
2
1
But according to our assumption A A* P P1 P
2
Again A A* P1 Q1 P1 Q1 2Q1.
A A*
Q1
2
A A*
But according to our assumption Q
2
Q1 Q
Hence P is Hermitian.
99
*
1
Again Q* A A*
2i
1 * 1 *
A A* A* A*
2i 2 i '
1
A* A ''
2i
1 *
A A as A' ' A
2i
1
A* A as A A
2i
1
A A* Q
2i
Q* = Q
Hence Q is Hermitian matrix.
So, A = P + iQ, where P and Q are Hermitian matrices.
Now it is required to prove that this representation is unique.
If possible, let A = P1 + iQ1 is another representation.
Where P1 and Q1 are Hermitian and P1 P, Q1 Q
* *
Now A* P1 iQ1 P1* iQ1
P1 Q1 as P1 Q1 are Hermitian
But A P1 iQ1
1
So, P1 A A*
2
1
But P A A*
2
So, P P1 Similarly Q Q1
100
Here our assumption is wrong
So, A = P + iQ is unique.
Adjoint of a matrix:-
Let A be a square matrix and B is a matrix whose elements the co-factors of
the corresponding elements of A, then the transpose matrix of B is called the Adjoint
matrix of A
Let Aij denote the cofactors of aij in the determinant |A| where A= [aij]
Then the transpose of [Aij] is the adjoint of A
a11 a12 a13
A a21 a22 a23
a31 a32 a33
2 0 1 0
Hence a11 8, a12 4
1 4 3 4
1 2 1 1
a13 6 7, a21 (4 1) 5
3 1 1 4
0 1 0 1
a22 3, a21 (3) 3
3 4 3 1
101
1 1 0 1 0 1
a31 2, a32 1, a33 1
2 0 1 0 1 2
8 5 2
Hence adjoint of the matrix A 4 3 1
7 3 1
Let A
ai1 ai 2 ai 3 ain
anl an 2 an 3 anm
Then (i,j) the element of the product A(adjA)= scalar product of the i th row of
A and jth column of adj A
I
o where ij 1
102
A where i j
Since of the elements of a row (or column) be multiplied by the cofactors of the
elements of the same row (or column) then the sum of the products will be
equal to the value of the determinant. But the elements of a row (or column) be
multiplied by the cofactors of parallel row (or column) then the product is
zero.
Thus in the product A, (adj A) only the diagonal elements exist and each equal
to |A| while other elements are zero.
A 0 0 0 .......... 0
0 A 0 0 .......... 0
A adj A
...............................................
0 0 0 0 .......... A
1 0 0 0
0 1 0 0
A 0 0 1 0
........................................
0 0 0 1
103
We have to prove that adj (AB) = (adj B). (adj A).
Now, AB. (adjB). (adjA).
= A. (B. adj B). adj A by Associative law
= A. | B| I. adj A
= | B|. (AI). adj A
= | B|. (A. adj A)
= | B|. | A| I.
= | A| | B| |I . =| AB| I.
Again, (adjB. adjA). AB
= adj B. (adj A. A). B
= adj B. | A| I . B. = | A|. adj B. (IB)
= | A|. adj B. B = | A| | B| I.
= | AB| I.
Thus AB(adjB). adjA=|AB| I = (adjB.adjA)AB.
So from (3) adj AB = adjB. adjA
Inverse of a matrix:-
If A and B are two matrix such that AB=BA=I then B is
called the inverse matrix of A and is denoted by A-1.
adjA
If we chosse B . then
A
adjA A.adjA AI
AB A I.
A A A
adjA
So B is the inverse of A
A
adjA
Hence A 1
A
104
(i) Prove that the necessary and sufficient condition for the existence of the
inverse of a square matrix is that it must be non-singular
Proof: The condition is necessary
Let A be an n n matrix and B be its inverse
Then by definition AB=BA=I
| AB| = | I|.
| A| | B|= I.
Hence . | A| 0. So the matrix A is non-singular.
The condition is sufficient
Let A be a non-singular n n matrix
Then | A| 0.
adjA
Let the matrix B
A
adjA A.adjA AI
Then AB A I
A A A
adjA A. A AI
Again BA A I
A A A
AB=BA=I
adjA
Hence B is the inverse of A.
A
105
or C=B
So our assumption is wrong thus the inverse in unique.
(iii) If A and B be two non-singular matrices of the same order then prove that
(AB)–1= B–1 A–1
Proof:- Let A and B be two non-singular matrices
then | AB| = | A| | B|
Since | A| 0 and | B| 0
So | AB| 0
Hence the non-singular matrix AB must possess on inverse.
Let A–1 and B–1 be the inverse of matrices of A and B respectively, then by the
definition of inverse of a matrix.
A A–1 = A–1A = I
and B B–1 = B–1B = I
When I is the unit matrix of order n
Also AI = IA = A
And BI = IB = B
We have to prove that (AB)–1 = B–1 A–1
Now, AB B 1 A 1 A BB 1
A 1
by Associative law
AIA 1
AA 1 I
Again, B 1A 1 AB B 1
BB 1
B
B 1I B
B 1B I
Thus , we get
AB B 1 A 1 B 1A 1 AB I
106
(iv) Prove that the adjoint of the transpose is equal to the transpose of the adjoint
matrix.
Proof:- Let A be a square matrix whose transpose matrix is A’. Then by the definition
of determinant.
| A| = | A’|
Again by the property of adjoint matrix, we have
(adj A ') A ' A '(adj A ') A' I AI …………………(1)
A ' ' adj A ' ' adj A ' ' A ' ' AI
Orthogonal matrix:-
A square matrix A is said to be orthogonal if A’A =I where A’ is the transpose
of A and I is the unit matrix with proper dimension
Properties of orthogonal matrix
(i) Prove that every orthogonal matrix is non-singular.
Proof:- Let A be a square matrix which is orthogonal then by the definition of
orthogonality
A’A = I …………………(1)
Where A’ is the transpose of A and I is the unit matrix of the same order as A.
107
Now, we know that
A' A …………………(2)
A A 1
2
A 1
or A 1
So A 0
Hence A is non-singular
(ii) To prove that I is an orthogonal matrix
Proof:- Evidently I’ = I
Hence I’I = I.I = I.
So I is orthogonal.
(iii) To prove that the product of two matrices of order n n is an orthogonal
matrix of order n n.
Proof:- Let A and B be two square matrices of order n n which are orthogonal
Then by definition, we have
A' A 1 …………………(1)
and B'B 1 .. … ……………(2)
Where A’ and B’ are the transpose matrices of A and B respectively and I is
the unit matrix of order n.
It is to prove that AB is orthogonal.
ie. AB ' AB I
108
B'I B from (1)
B'B
I
109
A’ = (A’A)B
A’ = IB
A’ = B
AB = AA’
Also (A’)’ = A
So AB = AA’
I = AA’
I = (A’) A’
(A’)’ A’ = I
So A’ is orthogonal.
Worked out Examples:-
3 4
2 1 2
(1) If A 1 1 B
1 3 4
2 0
2 3 4
1
AB 9 4 2
10 6 4
2 1 2 3 4
3 1 2
Again B’A’ 1 3 9 4 2
4 1 0
2 4 10 6 4
110
1 2 2
1
Solution:- Let A 2 1 2
3
2 2 1
1 2 2
1
Then A ' 2 1 2
3
2 2 1
1 2 2 1 2 2
* 1
Now A A ' 2 1 2 2 1 2
9
2 2 1 2 2 1
9 0 0 1 0 0
1 1
0 9 0 .9 0 1 0
9 9
0 0 9 0 0 1
1 0 1
Solution:- Let A 3 4 5
0 6 7
111
1 2 1
4 5 2
2 4 1
1 4 2
So adjoint of matrix = 2 5 4
1 2 1
0 1 1
(5) Find the inverse of the matrix A = 1 2 0
3 1 4
Solution:- Here | A | = 0 – 1 (4 + 1) + 3 = – 11 0
So A is non Singular
Co-factors of 1st row = 8, –4, –7
Co-factors of 2nd row = –5, –3, 3
Co-factors of 3rd row = –2, 1, –1
8 4 7
Matrix formed by the cofactors of A 5 3 3
2 1 1
8 5 2
Adj nt of A 4 3 1
7 3 1
8 5 2
AdjA 1
Inverse of A 4 3 1
A 11
7 3 1
8 5 2
11 11 11
4 3 1
11 11 11
7 3 1
11 11 11
112
4 3 3
A 1 0 1
4 4 3
3 3 4 3 4 3
Cofactors of 2nd row are 3, 0, 4
4 3 4 3 4 4
3 3 4 3 4 3
Cofactors of 3rd row are 3, 1, 3
0 1 4 1 1 0
4 1 4
So matrix formed by the cofactors A 3 0 4
3 1 3
4 3 3
Adjoint of A 1 0 1
4 4 3
4 3 3 4 3 3
1 adjA 1
A 1 0 1 1 0 1
A 1
4 4 3 4 4 3
Exercise
1. Find the transpose of Matrix
1 2 3
4 6 7
5 8 9
113
1 2 1 0 1 2
(i) 5 2 3 (ii) 1 2 3
1 1 2 1 1 1
1 1 1
3. 2 1 0 find A2 and show that A2 = A–1.
1 0 0
1 1 i 2
4. Prove that the matrix A 1 i 3 0 is Hermitian.
2 i 0
1 1 i 2 3i
5. If A 1 i 2 2
2 3i i 0
Answers:-
1 4 5
1. 2 6 8
3 8 9
114
1 7 3 1 7 3
1
2. (i) 3 1 1 , 3 1 1
10
4 2 8 4 2 8
1 1 3
(ii) 2 2 2 , A–1 Not exists
1 1 1
1 1 1 1 1 1
1
8. (i) 8 6 2 8 6 2
2
5 3 1 5 3 1
1 3 2 1 3 2
1
(ii) 3 9 4 , 3 9 4
2
3 5 2 3 5 2
RANK OF A MATRIX
Introduction
In a application of matrices to linear equations, an important idea is that of the
rank of the matrix.
Hence in this chapter the concept of the rank of a matrix will be developed.
Elementary transformations, that is, elementary operations of a matrix will also be
dealt with.
Minor of a Matrix
If from a matrix A of order m n, m – r rows and n – r columns are removed,
we are left with a square matrix of order r. The determinant of this matrix with a
positive or negative sign is called the minor of A and this minor is of order r.
a b c
Example. Let A d e f .
l m n
115
It is a 3 4 matrix.
Let us omit one (3 – 2) row (say, the first row) and two (4 – 2) columns (say,
f
first and second columns) from the matrix A; then we get a square matrix of
n
order 2.
The determinant
f f
or Is called the minor of A of order 2.
n n
116
(e) The rank of a non-singular matrix A of order n n is
n as its minor of order n = | A | 0.
(f) The rank of a matrix A = rank of the transpose matrix
A’.
Example. Let us consider a matrix
1 2 3
A .
4 5 6
1 2
Let us take any one of these minors, say .
4 5
117
Note. The order of the matrix remains unaltered under the elementary
transformations.
Notations : (a) Rij stands for the interchange of ith row and
jth row.
(b) Cij stands for the interchange of ith column and jth column.
(c) kRi stands for the multiplication of ith row by k ( 0).
(d) kCi stands for the multiplication of ith column by k ( 0).
(e) Ri + kRi stands for the addition of k times the jth row to the ith row.
(f) Ci + kCj stands for the addition of k times the jth column to the ith
column.
(g) R transformation stands for the elementary row transformation.
(h) C transformation stands for the elementary column transformation.
Equivalent Matrices
Two matrices are said to be equivalent if one is obtained from the other by a
finite number of elementary transformations. If the matrix B is equivalent to the
matrix A, then it is written as B A.
The following results are crystal clear –
(a) A A.
(b) A B. B A.
(c) A B, B C. A C.
Elementary Matrices
It is a particular class of equivalent matrices.
A matrix obtained from a unit matrix by some definite number of elementary
row (or column) transformations is called an elementary row matrix (or elementary
column matrix).
An elementary matrix is denoted by E-matrix. There are four different types of
elementary matrices, viz.,
118
(i) Eij : It stands for the matri x obtained from a unit
matrix by interchanging its ith and jth rows (or columns).
(ii) Ei(k) : It stands for the matrix obtained from a unit
matrix by multiplying the elements of its ith row (or columns) by non-
zero scalar k.
(iii) Eij(k) : It stands for the matrix obtained from a unit
matrix by adding the product of a non-zero scalar k with the jth row (or
column), to the ith row (or column).
(iv) Eij(k) : It stands for the transpose of Eij(k).
1 0 0
Examples. Let I 3 0 1 0 .
0 0 1
1 0 0
0 0 1 , (interchanging first and second rows)
0 1 0
1 0 0
(ii) 0 2 0 , (multiplying R2 or C2 by 2)
0 0 1
1 0 3
(iii) 0 1 0 , (adding 3 R3 to R1)
0 0 1
1 0 0
4 1 0 , (adding 4 C2 to C1).
0 0 1
119
1 0 0 1 0 3
(iv) 0 1 0 , which is the transpose of 0 1 0 .
3 0 1 0 0 1
Theorems
I. (i) Every elementary row transformation on the
product AB of two matrices A and B is equivalent to the same
elementary row transformation on the pre-factor A of AB.
(ii) Every elementary column transformation on the
product AB of two matrices A and B is equivalent to the same
elementary column transformation on the post-factor A of AB.
Proof. Let A and B be two matrices of orders m n and n p respectively so
that the product AB is defined.
Let the rows of A be denoted by
R1, R2, R3,……, Rm
While the columns of B are denoted by
C1, C2, C3,……, Cp.
R1
R2
Then we have A ..... and B = [C1, C2……. Cp].
.....
Rm
120
R1
R2
AB ..... [C1, C2……. Cp]
.....
Rm
121
A = AI,
If u denotes any elementary column transformation,
Then u (A) = u(AI)
= A (uI)
= AE .. (2)
where E is the elementary matrix corresponding to the elementary column operation
u.
From (1) and (2), we arrive at conclusion that each elementary row (or column)
operation on a matrix is equivalent to pre (or post) multiplication by the
corresponding elementary matrix.
122
We have to prove that t = r.
Since the rank of A is r, therefore, by the definition of rank of a matrix every
minor of order (r + 1) of A is zero.
Let | B0 | be a minor of B of order (r + 1) and | B0 | is the minor of A, of order (r
+ 1), obtained in the same way as | B0 | is obtained.
| A0 | = 0.
The following cases arise.
CASE I. If | B0 | does not contain the ith row, then
| B0 | = | A0 | = 0, as | A0 | = 0.
CASE II. If | B0 | contain both the ith and the jth row, then perform the
operation [ith row – k (jth row)] on | | B0 |.
CASE III. If | B0 | contain the ith row but does not contain the jth row, then
| B0 | = | A0 | + k [some minor of order (r + 1) of A]
=0+k 0
= 0.
Hence every minor of order (r + 1) of B is zero.
rank of B ≤ r.
t ≤ r. .. (1)
Now A can also be obtained from B by an elementary transformation Ri + ( –
k) Rj.
We can deduce, as above, that
r ≤ t. .. (2)
Hence, from (1) and (2), we find that t = r.
Thus elementary transformation do not alter the rank of a matrix.
Cor. 1. The pre-multiplication (or post-multiplication) by any elementary
matrix does not change the rank of a matrix.
123
We know that every elementary row (or column) transformation of a matrix
can be brought about by pre-multiplication (or post-multiplication) with the
corresponding elementary matrix. Hence the above theorem gives the corollary.
Cor. 1. The pre-multiplication (or post-multiplication) by any series of
elementary matrices does not alter the rank of a matrix.
This follows from the cor. 1.
Every non-zero matrix A of rank r can be reduced to one of the following forms :
Ir 0 Ir
, , Ir 0 , Ir
0 0 0
Method for finding the rank of a matrix with the help of the normal form of the
matrix.
Reduce the given matrix by elementary transformations to
the normal form.
Let the given matrix be reduced to the normal form
Ir 0
.
0 0
Ir 0
The rank of this matrix . is r.
0 0
Hence the rank of the given matrix is also r, as the rank of a matrix remains
unaltered under elementary transformations.
124
If the elementary transformations reduce the given matrix to a matrix, whose
rank can easily be obtained by the definition of rank, then we do not reduce the given
matrix to normal form.
WORKED OUT EXAMPLES
1. (i) Find the rank of the matrix.
1 2 3 4
A 2 4 6 8
1 2 2 4
Solution. Obviously the highest order of all the minors that can be formed from
the matrix A is 3.
Here all the minors of order 3 are.
1 2 3
(i) 2 4 6 0
1 2 2
1 2 4
(ii) 2 4 8 0
1 2 4
1 3 4
(iii) 2 6 8 0
1 2 4
2 3 6
(iv) 4 6 8 0
2 2 4
We find that all minors of order greater than 2 vanish and at east one minor of
order two is not zero.
125
Hence, by the definition of the rank, (A) = 2.
1. (i) Find the rank of the matrix.
14 18 6 8 10
6 125 21 9 25
.
7 9 3 4 5
35 45 15 20 25
14 18 6 8 10
6 125 21 9 25
Solution. Let A .
7 9 3 4 5
35 45 15 20 25
Thus every determinant picked out of it, of order 3, 4 will be zero. But every
determinant of order 2 is not zero. Hence the rank of the matrix is 2.
2. Find the rank of
1 1 1 1 2 1
2 3 4 6 12 6 .
3 2 3 3 10 5
1 1 1 2 1 1
2 6 3 12 4 6 ,
3 5 2 10 3 5
as the operation of addition is defined
126
0 1 1
8 9 10 .
8 8 8
3. Prove that the points (x1, y2), (x2, y2) and (x3, y3) are collinear if and only if
x1 y1 1
the rank of the matrix x2 y2 1 is less than 3.
x3 y3 1
Solution. Let the given points (x1, y1), (x2, y2) and (x3, y3) lie on the straight line
ax + by + c = 0.
Then ax1 + by1 + c = 0, .. (1)
ax2 + by2 + c = 0, .. (2)
ax3 + by3 + c = 0, .. (3)
127
Eliminating a, b and c from (1), (2) and (3), we get
x1 y1 1
x2 y2 1 0. .. (4)
x3 y3 1
x1 y1 1
Let A x2 y2 1 .
x3 y3 1
x1 y1 1
| A| x2 y2 1
x3 y3 1
= 0, by (4).
Hence (A) < 3.
Conversely, let (A) < 3, then | A | = 0
x1 y1 1
x2 y2 1 0.
x3 y3 1
the points (x1, y2), (x2, y2) and (x3, y3) are collinear.
4. Find the rank of the matrix
2 3 1 1
1 1 2 4
.
3 1 3 2
6 3 0 7
128
Let us consider a minor of order three of the matrix B.
1 0 0
0 5 3 =5 9–4 3 = 45 – 12 = 33 0.
0 4 9
1 0 0 0
5 3 7
0 5 3 7
Now 4 5 10
0 4 9 10
9 12 17
0 9 12 17
5 3 7
4 9 10 ,
0 0 0
operating R3 –( R1 + R2)
= 0.
Hence (A) = 3.
129
5. Reduce the matrix.
1 1 2 3
4 1 0 2
A
0 3 1 4
0 1 0 2
Solution.
Hence [I4 . 0] is the normal form of A and the rank of the matrix A is 4.
6. Find the rank of the matrix.
2 3 1 1
1 1 2 4
A
3 1 3 2
6 3 0 7
by reducing it to triangular form.
130
Obviously the rank of the matrix cannot be equal to 4, it
can be at most equal to 3.
1 0 0 0
A~ 0 5 3 7 .
0 4 9 10
Hence (A) = 3.
EXAMPLES 8
1. Find the rank of
1 3 4 1 2 3
(i) . (ii) .
2 6 8 2 4 3
131
1 2 3
(i) 4 5 6 .
2 1 2
1 2 3
[Hint. Minor of order 3 is 4 5 6 0.]
2 1 2
1 2 3
(ii) 2 4 7 .
3 6 10
1 3 4 3
(iii) 3 9 12 9 .
1 3 4 3
1 1 1
(iv) a b c , a , b , c being real.
a2 b2 c2
2 4 5
(v) 4 9 10 .
6 12 15
0 i i
(vi) i 0 i .
i i 0
where i2 = – 1
3. Obtain the rank of the matrix
1 1 2 0 1 2
3 4 2 4 6 2
.
2 3 6 0 1 2
1 4 8 2 2 17
0 1 0 0
0 0 1 0
4. If U .
0 0 0 1
0 0 0 0
132
prove that (U) = 3, (U2) = 2.
( U3) = 3, (U4) = 0.
1 0 0
[Hint. | U | = 0. But 0 1 0 0.
0 0 1
(U) = 3.
Find U2, U3, U4, and then obtain their ranks.
5. (i) Obtain the rank of the matrix
1 1 1 1
1 3 2 1
A
2 0 3 2
3 3 3 3
[Hint, Apply successively C21 (– 1), C31 (– 1), C41 (– 1),
R21 (– 1), R31 (– 2), R41 (– 3).]
(ii) Find the rank of the following matrix–
6 1 3 8
4 2 6 1
.
10 3 9 7
16 4 12 15
2 1 4 7
(iii) Find the rank of the matrix 3 6 2 1 .
0 0 1 5
1 3 4 5
(iii) Find the rank of 1 2 6 7 .
1 5 0 10
1 2 1 3
(iv) Find the rank of 2 4 4 7 .
1 2 1 2
133
6. (i) Reduce the matrix
0 1 2 1
A 1 2 3 2
3 1 1 3
to triangular form by elementary row operations and hence find its rank.
Answers
1. (i) 1. (ii) 2.
2. (i) 3. (ii) 2. (iii) 1.
(iv) 3, when a b c;
2, when a b c, that is, only any two are equal;
1, a b c. (v) 1. (vi) 2.
3. 2. 5. (i) 3. (ii) 2. (iii) 3. (iv) 3.
0 1 0 0
6. (i) Normal form of A is 0 1 0 0 ; (A) = 3.
0 0 1 0
(ii) 2.
1 1 2 0
7. 0 1 2 1 ; (A) = 3.
0 0 12 4
134
Matrix array Number Symbol expressions
4 6 3
Matrix
2 4 6 2 3.
Matrix size
135
Square Matrix A invertible Matrix B exist
AB = BA = I. Matrix B A inverse A–1
denote
136
CHAPTER - 4
LINEAR PROGRAMMING
Objectives
Unit
Convex Sets
Convex Sets properties
Solution of LPP by Graphical Method
Solution of LPP by Simple Method
137
Example :-
Show that the set
S = {(x1, x2) : 3x12 + 2x22 ≤ 6} is convex
Solution :-
Let x, y S where x = (x1, x2) and y = (y1, y2)
Then the line segment joining x and y is the set
{u : u = x + (1 + ) y, 0 ≤ ≤ 1}
For some . Let u = (u1, u2), be a point in the set
So that u1 = x + (1 + ) y1 and u2 = x2 + (1 + ) y2
Now 3u12 + 2u22 = 3{ x1 + (1 – ) y1}2 + 2 { x2 + (1 –
)y2}2
2
= (3x12 + 2x22) + (1 – )2 (3y12 + 2y22)
+ 2 (1 – ) (3x1y1 + 2x2y2)
2
≤6 + 6(1 – )2 + 12 (1 – )
2 2 2 2
Since 3x1y1 + 2x2y2 ≤ 3x1 2 x2 3 y1 2 y2
138
= (Cx1) + (1 – ) Cx2
= K + (1 – ) K = K
Therefore the point x1 + (1 – )x2 where 0 ≤ ≤ 1 lies in the hyperplane.
Hence the hyperplane is a convex set.
Theorem 2 : The closed half spaces H1 = (x : CX K} and H2 = (x : CX ≤ K} are
convex sets.
Let x1, x2 be any two points of its. Then Cx1 K and Cx K.
If 0 ≤ ≤ 1 then
C{ x1 + (1 – )x2} = C x1 + C (1 – )x2
= {Cx1) + (1 – ) K = K
Therefore, x1, x2 H1 and 0 ≤ ≤1
x1 + (1 – )x2 H1
Hence H1 is a convex set.
Similarly it can be proved by replacing by ≤ the H2 is a convex set.
Corl: Also the open half spaces H1 ={X : Cx > k}
and H2 ={X : Cx < k} are convex sets.
Theorem 3 : Prove that the sphere is a convex set
Proof:-
For simplicity we take the equation of the sphere as x2 + y2
+ z2 = a2
Let P(x1, y1, z1) and u2 = (x2, y2, z2)
Since P and Q are any two points of the set representing the
sphere.
Therefore x12 + y12 + z12 ≤ a2 and x22 + y22 + z22 ≤ a2 ……(1)
The line joining u1 and u2 is given by
u = u1 + (1 – ) u2. 0 ≤ ≤1 ……(2)
Let u be represented by u = ( , , )
139
Then = x1 + (1 – ) x2
= y1 + (1 – ) y2
= z1 + (1 – ) z2
Thus the co-ordinates of any point on the line segment joining (x1, y1, z1) and
(x2, y2, z2) are given by (3).
2 2 2
We have to prove that + + < a2
2 2 2
We have to prove that + + < a2
We have,
2 2 2
+ + = { x1 + (1 – )x2}2 + (1 – )y2}2
+ { z1 + (1 – )z2}2
2
= (x12 + y12 + z12) + (1 – ) (x22 + y22 + z22)
+ 2 (1 – )2 (x1x2 + y1y2 + z1z2)
by Cauclty‟s in equality
2 2
a + (1 – )2 a2 + 2 (1 – ) a2
= a2 { 2
+ (1 – )2 + 2 (1 – )}
= a2 { 2
+ (1 – )2 = a2
2 2 2
Thus + + < a2
This shows that any point on line segment belong to the set
But the point is arbitrary. Hence the line segment represented by (2) belongs to
the set.
Thus a sphere is a convex set.
Theorem 4 : The intersection of two convex sets is also a convex
set.
Proof:-
Let A and B be two convex sets and let X = A B
It is required to prove that X is a convex set.
140
Let x1, x2 X and S = {x : x = x1 + (1 – )x2, 0 ≤ ≤ 1}
Now x1, x2 X x1, x2 A
S A [ A is a convex set ]
Also x1, x2 X x1, x2 B
S B [ B is a convex set ]
x1, x2 X S A and S B
S A B
S X
Hence S is a convex set
141
i) If the straight line does not pass through the origin, then we substitute (0, 0) in
the given inequation.
If the given inequation is satisfied, then that member of the half planes
represents the solution set of the given inequation which contains the origin.
If the co-ordinates of origin (0, 0) do not satisfy the inequations, then that
member of the half planes represents the solution set which does not contain
the origin.
ii) If the st. line passes through the origin, then take any point ( , 0) on the x-axis
and determine the solution set of the given inequation as in (i).
Example 1 :- Y
Find the solution set of the inequations
2x + 3y ≤ 6, x 0. y 0 B(0, 2)
2x+3y=6
Solution :- A(3.0)
The corresponding equation is O
x y
2x + 3y ≤ 6, i.e. 1
3 2
The straight line cuts x-axis at A(3.0) and y-axis at
B(0, 2). Thus the line is represented in the graph
by AB. For the solution set we put (0, 0) in the
given inequation we find that 2 0 + 3 0 ≤ 6.
which is true. Hence solution set is the half plane
which contains the origin. This is shown by the
shaded region in the graph. If we apply further
condition x 0. y 0 that restricts the solution to
the first quadrant.
Only then the solution set will be only region OAB as shown in fig ii.
Example 2 :-
Find the solution set of the following inequation
142
Solution :-
The corresponding equation 3x + 4y = 12
x y
i.e. 1
4 3
The line intersects x-axis at (4, 0) and y-axis
at (0, 3). So the line AB represents the Y
equation Putting the co-ordinates of the
origin 0 (0, 0) in the inequaiton we find that
3 0+4 0 12 which is not true. B B(0,3)
Hence (0, 0) is not a member of the solution
Set of the inequation. Thus the solution set 3x+4y=12
of the given inequation is that half plane X
Which does not contain O (0, 0) O(0,0) A(4,0)
143
Proof :-
Let us prove the following result first.
Let P = (x1, x1) and Q = (x1‟, x1‟) S
144
Let us suppose that at the extreme point P the function P
Q
takes on a value greater than or equal to the value at
any of other extreme point and at the extreme point Q,
A
the function takes on the smallest value . Thus the
value of the function at any other extreme point must
S R
lie between and .
M
It is sufficient to prove that the value of the function at L
any interior point say A lies between and , so that the optimum value is achieved
always at the extreme point.
Draw a straight line joining P and A produce it to meet the edge of the polygon at M
which joins the extreme point L and R.
As proved earlier, the value of the function at M must lie between its values at L an R.
Since each of the values of the function at L and R lies between and .
Now the value of the function at P is and the value of the function at M lies between
and . Since A lies on the line segment PM, therefore the value of the function at A
lies between the value of the function at P and the value of the function at M, i.e. the
value of the function at A lies between and a value that lies between and i.e. the
value of the function lies between and . But A is any interior point. This means
that the value of the function at any interior point lie between and . Thus the
optimum value of the function is obtained at the extreme points only.
145
Step2. Set up the mathematical formulation of the problem.
Step3. Plot a graph representing all the constraints of the problem.
To plot the graph, convert all inequations into equations. Then identify the
feasible regions (solution space). The feasible region is the intersection of all
the regions represented by the constraints of the problem and is restricted to
the first quadrant only.
Step4. The feasible region obtained in (3) may be bounded or unbounded compute
the co-ordinates of the corners points of feasible region.
Step5. Find out the value of the objective function at each corner point determined
in step 4.
Step6. Such that corner point that optimizes (maximizes or minimizes) the value of
the objective function. It gives the optimum feasible solution.
Worked Out Examples
Example 1 :- Solve the following linear programming problem
graphically
Max. Z = 5x1 + 3x2
Subject to the following constraints.
3x1 + 5x2 ≤ 15
5x1 + 2x2 ≤ 10
x1 + x 2 0
Solution :-
Let OX and OY represent the co-ordinate
axis with O as origin.
Firstly, we observe that any point (x1 + x2)
in the plane which satisfies the condition.
x1 0, x2 0 lies in the first quadrant.
Thus our solution is available in the first
quadrant only.
146
Now, consider the constraints by converting the inequalities into
equations.
3x1 + 5x2 = 15
5x1 + 2x2 = 10
First consider the equation 3x1 + 5x2 = 15.which represents a straight line. This cuts
X-axis at the point (5, 0) and Y-axis at (0, 3). In general there are two methods to find
x1 x2
the intercept on the axis convert the equation in the intercept form i.e. 1
5 3
which indicates that the intercept on X and Y axis are 5 and 3 respectively. In other
method, put y = 0 to get intercept in X-axis and x = 0 to get intercept on Y-axis.
Hence we plot the line 3x1 + 5x2 = 15 as a straight line AB which cuts X-axis at A (5,
0) and Y-axis at B(0, 3).
Similarly 5x1 + 2x2 = 10 is the equation of a straight line CD which cuts X-axis at the
point C(2, 0) and Y-axis at the point D(0, 5). So plot the line CD also as shown in the
figure.
Now, we have to determine the feasible region of the inequations.
3x1 + 5x2 ≤ 15
and 5x1 + 2x2 ≤ 10
for this, we take the origin O(0, 0) and examine which side
of the given straight line the origin lies.
147
Thus the required feasible region OCPB which is a convex set.
Next, we determine the co-ordinates of the extreme points O. C. P.B of the convex set
OCPB.
The co-ordinates of O obviously are (0, 0)
The co-ordinates of C are obtained by solving
5x1 + 2x2 = 10 and x2 = 0
Hence they are (2, 0).
The co-ordinates of B are similarly obtained by 3x1 + 5x2 = 15 and x1 = 0 and hence
they are (0, 3).
The co-ordinates of P are obtained by solving 3x1 + 5x2 = 15 and 5x1 + 2x2 = 10 and
20 45
hence the co-ordinates of P are ,
19 19
Finally calculating the value of Z = 5x1 + 2x2 for the points O. C. B. P we have.
Z(O) = 5 0+3 0=0
Z(C) = 5 2+3 0 = 10
Z(B) = 5 0+3 3=0
20 45 100 135 135 7
Z(P) = 5 3 12
19 19 19 19 19 19
Thus the maximum Z is
7
attained at the point P and it is 12 .
19
Example 2 :- Solve the following linear programming problem
graphical
Max. Z = 4x1 + 7x2
Subject to the constraints
x1 + 2x2 ≤ 20
x1 + x2 ≤ 15
x2 ≤8
x1 0, x2 0.
148
Solution :-
Let OX and OY represent the co-ordinate
axis with O as origin.
Firstly, we observe that any point (x1, x2) in
the plane which satisfies the condition. x1
0, x2 0 lies in the first quadrant only.
Next, we convert the inequaitons in
equalities and plot the graph of the lines
x1 + x2 = 20, x1 + x2 = 15. x2 = 8
Clearly, the above equations represent the straight line the first line
x2 + 2x2 = 20 cuts X-axis at the point A(20, 0) and Y-axis at the
point B(0, 10)
the straight line x2 + x2 = 15 cuts x-axis at the point C(15, 0) and Y-axis at the point
(0, 15) the straight line x2 = 8 is parallel to x-axis and it cuts
Y-axis at the point E(0, 8)
Their graphs are given above
clearly (0, 0) origin satisfies each of the inequaitons
x1 + 2x2 ≤ 20
x1 + x2 ≤ 15
x2 ≤8
We find the feasible region of the L.P. problem is the intersection of all the feasible
regions of the inequaitons which contain the origin.
The shaded portion OCPQE is the feasible region which is clearly a convex set. Now,
we determine the co-ordinates of the extreme points O. C. P. Q. E of the convex set.
The co-ordinates of O all obviously (0, 0)
The co-ordinates of C are (15, 0)
The co-ordinates of E are (0, 8)
The co-ordinates of P are obtained by solving the equations
x1 + 2x2 = 15
149
and x1 + x2 = 20
The co-ordinates of P thus obtained are (10, 5)
The co-ordinates of Q are obtained by solving the equations
x1 + 2x2 = 20 and x2 = 8
Hence we obtain x1 = 4, x2 = 8. Thus the co-ordinates of Q are (4, 8).
Now, we calculate the value of Z = 4x1 + 7x2 for the extreme points O. C. P. Q. E.
Z(O) = 4 0+7 0=0
Z(C) = 4 15 + 7 0 = 60
Z(P) = 4 10 + 7 5 = 40 + 35 = 75
Z(Q) = 4 4+7 8 = 16 + 56 = 72
Z(E) = 4 0+7 8 = 0 + 56 = 56
Therefore Max Z is attained at the point P and its value is 75.
Example 3 :-
Solve the following linear programming problem by graphical method
Maximize Z = 2x + 5y
Subject to the constraints
x1 + 4y ≤ 24
3x + y ≤ 21
x +y≤9
x, y 0
Solution :-
Let OX, OY represent the co-ordinate
axis with O as origin. Clearly, any
point (x, y) in the plane satisfying the
constraints x, y 0 lies in the first
quadrant.
Next, convert the inequaitons of
constraints into equations.
150
x + 4y = 24
3x + y = 21
x +y=9
Draw the graphs of these equations which represent straight lines.
The equation x + 4y = 24 represents a straight line which cuts X-axis
at A(24, 0) and Y-axis at B(0, 6).
The equation 3x + y = 21 cuts X-axis at C(7, 0) and Y-axis at D(0,
21).
Finally the line x + y = 9 cuts X-axis E(9, 0) and Y-axis at F(0, 9).
Since the origin satisfies each of the inequaions
x + 4y ≤ 24
3x + y ≤ 21
x +y≤9
We find the feasible region of the L.P. problem is the intersection of all the feasible
regions of the inequaitons which contain the origin.
This is given by the shaded region OCQPB.
Next we determine the co-ordinates of extreme points O, C, Q, P, B which is a convex
set
The co-ordinates of O are (0, 0)
The co-ordinates of C are (7, 0)
The co-ordinates of Q are obtained by solving the equaitons (0, 8)
3x + y = 21
and x +y=9
Hence we obtain x = 6, y = 3 i.e. the co-ordiantes of Q are
(6, 3).
The co-ordinates of P are obtained by solving the equations
x + 4y = 21
and x +y=9
151
Hence we obtain x = 4, y = 5 i.e. the co-ordiantes of P are
(5, 4).
The co-ordinates of B are (0, 6)
Now, we calculate the value of Z = 2x + 5y for the points O, C, Q, P and B.
We have
Z(O) = 2x + 5y = 2 0+5 0=0+0=0
Z(C) = 2x + 5y = 2 7+5 0 = 14 + 0 = 0
Z(Q) = 2x + 5y = 2 6+5 3 = 12 + 15 = 27
Z(P) = 2x + 5y = 2 4+5 5 = 8 + 25 = 33
Z(B) = 2x + 5y = 2 0+5 6 = 0 + 30 = 30
Therefore Max. Z is attained at the point P(4, 5) and its value is 33.
Example 4 :-
Solve the following linear programming problem by graphical method
Max. Z = 5x + 7y
Subject to the following constraints
x+ y≤4
3x + 8y ≤ 24
10x + 7y ≤ 35
x, y 0
Solution :-
Let OX and OY represent the co-
ordinate axis with O is the origin.
Firstly, we observe that any point (x,
y) in the plane satisfies the condition.
x 0. y 0 lies in the first quadrant.
Thus the solution of our problem lies
in the first quadrant only.
152
Now, consider the inequations as equation and draw the graph of
these equations.
x+ y=4
3x + 8y = 24
10x + 7y = 35
We observe that
(i) The straight line x + y = 4 cuts the x-axis at the point A(4,
0) and the y-axis at the point B(0, 4).
(ii) The straight line 3x + 8y = 24 cuts the x-axis at the point C(8, 0) and the y-axis at the
point D(0, 3).
(iii) The straight line 10x + 7y = 35 cuts the x-axis at the point E(3.5.0) and the y-axis at
the point F(0, 5).
The graphs of these equations i.e. of the lines AB. CD, EF are given in the figure
above.
Since the origin satisfies each of the inequaitons
x1 + x2 ≤ 4
3x1 + 8x2 ≤ 24
10x1 + 7x2 ≤ 35
We find the feasible region of the L.P. problem is the intersection of all the feasible
regions of the inequaitons which contain the origin. This is given by the shaded region
OEPQD.
Now we determine the co-ordinates of extreme points O, E, P, Q, D of the
convex set
The co-ordinates of O are (0, 0)
The co-ordinates of E are (3.5.0)
The co-ordinates of D are (0, 3)
The co-ordinates of Q are obtained by solving the equaitons
x + y = 4 and 10x + 7y = 35
153
7 5 7 5
Which we obtain x = , y = , i.e. the co-ordinates of P are ,
3 3 3 3
The co-ordinates of Q are obtained by solving the equations
x+y=4
and 3x + 8y = 24
8 12 8 12
Which we obtain x = ,y= , i.e. the co-ordinates of Q are ,
5 5 5 5
Now, we calculate the value of Z = 5x + 7y for the points O, E, P, Q, D.
We have
Z(O) = 5 0+7 0 =0
Z(E) = 5 3.5 + 7 0 = 17.5
Z(D) = 5 0+7 3 = 21
7 5
Z(P) = 5 +7
3 3
35 35 70 1
= 23
3 3 3 3
8 12
Z(Q) = 5 +7
5 5
40 84 124
= 24.8
5 5 5
Therefore, Max. Z is attained at the point Q and its value is 24.8.
Simplex Algorithm :-
The following are the steps to find the optimal solutions.
Step1. Check whether the objective function of the given L.P problem is to be
maximized or to be minimized.
If it is to be minimized them we convert into a problem of maximizing by
using the fact that min / -max (- /) (theorem 8.12)
Step2. Check whether all b (i) 1.2…… m are non-negative. If any one of b is
negative then multiply the corresponding inequation of the constraints by
also that all b are non-negative.
154
Step3. Convert all the inequations of the constraints into equation by introducing
slack or surplus variables in the constraints. The coefficients of these
variables in the objective function will the put as zero.
Step4. Obtain an initial basic feasible solution to the problem in the form xB B 1b
where B is the basis sub-matrix.
Step5. Compute the quantities (called not evaluation) c - /) (i = 1.2.3 …….m) by
using the relation /1 CBy1 and y1 B 1 then examine the sign of e1 – zj
(i) If all e1 – zj ≤ 0, then the initial basic feasible solution\ is an eptumum basic
feasible solution.
(ii) If at least are cj – zj > 0, proceed on the next step.
Step6. If there more than one positive c j – zj then choose the max of them let it be c j
– zj for some j = k.
(i) If all yij ≤ 0, (i = 1.2.3 …….m) then there is an unbounded solution to the
give problem.
(ii) If at last one yik > 0 ( i 1.2.3 ……..m) then corresponding vector Y1 inters
the basic YB
XB
Step7. Compute the ratio min
y1 .yu 0, i 1.2.................m
Then the vector Yk inters the basis YB and the vector r is removed from the
basis matrix B.
Such vectors are usually called incoming vector and outgoing vector. The
incoming vector Yk is always selected corresponding to k for which Ck –Zk =
ma X(C1 Z1).
The outgoing vector r is selected according to minimum ratio rule for
predetermined value of k.
155
The common element Ykr which is the kth row and ith column is known as the
key element or pivotal element of the table.
Step8. Convert the key element to unity by dividing its row by key element itself and
all other elements in its column to zero by using the elementary operation.
Step9. Again compute the net evaluation C1 Z1 as in step 5 and the computations
procedure until either an optimal solution is obtained or there is an indication
of an unbounded solution.
Illustrative Example
Example 1:-
Use simplex method to solve the following L.P. problem
Max. Z , Zx1 + 5x2
Subject to x1 + 2x2 ≤ 6
4x1 + 3x2 ≤ 6
x1 x2 ≤ 0
Solution :-
(i) The given problem is that of maximizing the objective function.
(ii) All the b‟s are non-negative
(iii) The inequalities of the constrained are converted into equations by introducing
slack variables x1 and x2 as follows.
x1 + 2x2 + x3 = 6
4x1 + 3x2 + x4 = 12
Then the new objective function is
Z = 7x1 + 5x2 + 0, x3, 0 x4
Here C1 = 7, C2 = 5, C3 = 0, C4 = 0
The set of constraints in the matrix form can be written is
AX b
156
1 2 1 0
7 6
Where 4 3 0 1 . b
3 12
1 1 1 1
1 2 1 0
So, A = ( 1, 2, 3, 4) 4 3 0 1
1 1 1 1
Clearly the rank of matrix A is 2. Hence there are two linearly independent
column vectors of A.
1 0
Obviously 3 = and 4 = are two linearly independent column vector
0 4
of A.
3 4
1 0
So, we have B = as a non singular basis sub matrix of A.
0 1
x4 = (0, 0)
(v) The value of the objective function for this initial basic feasible solution x 4 0,
x2 = 0.
6
x3 = 6, x4 = 12 is Z CBXB = (0, 0) 0
12
To see whether there exists some better basic feasible solution, we compute Cj
–Zj corresponding to the non-basic variables x1 and x2 of course C3 – Z3 = 0
and C4 – Z4 = 0
157
We have Y1 = B–1 1 = 1
Z1 = CBY1 = (0, 0) 1 =0
7–0=7
Again Y2 = B–1 2 = 2 = 2
Z2 = CBY2 = (0, 0) 2 =0
C2 – Z2 = 5
Since Cj – Zj are positive, we have to proceed for the next step
Thus the intial simplex table is
C 7 5 0 0
CB YB XB Y1 Y2 Y3 Y4
( 1) ( 2) ( 1) ( 2)
0 Y3 x3 – 6 1 2 1 0
0 Y4 x4 – 12 4 3 0 1
Z = CB X B xj 0 0 6 12
0
Cj 7 5 0 10
Cj – Zj –7 –5 0 0
Incoming Outgoing
Vector vector
(vi) Since more than one Cj – Zj are positive. Therefore, no choose the maximum of
them viz 7 which this in the column Y1
Since all the components of Y1 are positive therefore, the vector Y1 (– 1) will
inter into the basis YB
(vii) To select the vector which should leave the basis YB.
X Br
We compute min
yr1
X B1 X B 2 6 12
= mi = min
y11 y21 1 4
158
X Br X B2
Or, = min {6, 3} = 3 =
yr1 y21
(viii) The key element or the pivotal element is Y21 = 4 which becomes the leading
element for the next iteration convert the key element to unity by dividing that
row by 4 and all other members of column Y1 = ( 1) to zero by the application
of suitable elementary transformations clearly dividing 2nd row by 4 and
subtracting the resulting elements from (–) row 1 get new set of elements. The
whole procedure are shown below :-
XB Y1 Y2 Y3 Y4
R1 x3 6 1 2 1 0
R2 x4 12 4 3 0 1
1
Applying R2, we get
4
XB Y1 Y2 Y3 Y4
R1 x3 6 1 2 1 0
3 1
R2 x4 3 1 0
4 4
1
Applying R2, we get
4
XB Y1 Y2 Y3 Y4
5 1
R1 x3 = 3 0 1
4 4
3 1
R2 x4 = 3 1 0
4 4
159
Thus the new simplex table is as given below.
B
XB Y1 Y2 Y3 Y4
CB
Y3 0 x3 = 3 0 5 1 1
4 4
Y1 7 x1 = 3 1 3 0 1
4 4
Z = CBXB xj 3 0 3 0
=7 3 Cj 7 5 0
= 21 Cj –Zj 1
0 0 0
4
It is from the table that all C j –Zj are negative and hence an optimal solution
has been achieved. Thus the optimum basic feasible solution to the L.P.
problem is x1 = 3.
Example 2 :-
Use the simplex method to solve the following linear programming problem
Max Z = 5x1 + 7x2
Subject to 2x1 + 3x2 ≤ 13
3x1 + 2x2 ≤ 12
x1 0, x2 0
Solution :-
The given LP problem is that of maximizing the objective
Function and all b‟s are non-negative.
Thus in the first step we have to introduce slack variables
X3 0, x4 0 such that the inequaitons of the constraints are converted into
equations
2x1 + 3x2 + x3 = 13
3x1 + 2x2 + x4 = 12
160
Then the new objective function is
Z = 5x1 + 7x2 + 0 . x3 + 0. x4
Here C1 = 5, C2 = 7, C3 = 0, C4 = 0
The set of equations can be written in the matrix form as
1 2 3 4
2 3 1 0
3 2 0 1
or, AX = b
x1
1 2 3 4
x2 13
Where A = 2 3 1 0 X
x3 12
3 2 0 1
x4
i.e. x3 = 13, x4 = 12
Thus initial basic feasible solution x1 = 0, x2 = 0, x3 = 13, x4 = 12
Also CB = CB , CB =(coefficient of x3 coefficient x4)= (0, 0)
1 2
The value of the objective function for this initial basic feasible
Solution x1 = 0, x2 = 0, x3 = 13, x4 = 12 is
13
Z = CBXB = (0, 0) =0
12
161
To see whether there exists some better basic feasible solution. We compute C2
– Z2 corresponding to the non-basic variables x1 and x2; of course C3 – Z3 = 0
and C4 – Z4 = 0
Yj = B–1 j = I2 j = j, j = 1, 2, 3, 4
2
Z1 = CBY1 = (0, 0) =0
3
C1 – Z1 = 5 – 0 = 5
2
Also Z1 = CBY2 = (0, 0) =0
3
C2 – Z2 = 7 – 0 = 7
1
Similarly Z3 = CBY2 = (0, 0) =0
0
C3 – Z3 = 0 – 0 = 0
0
And Z4 = CBY4 = (0, 0) =0
1
C4 – Z4 = 0 – 0 = 0
Thus the initial simplex table is
B CB XB Y1 Y2 Y3 Y4
( 1) ( 2) ( 1) ( 2)
Y3 0 x3 – 13 2 3 1 0
Y4 0 x4 – 12 3 2 0 1
Z = CBXyxj xj 0 0 6 12
Cj 7 5 0 10
Cj – Zj –7 –5 0 0
Incoming Outgoing
Vector vector
from the table, it is clear that there are two C4 – Z4 are positive thus the
improved solution possible.
162
We choose the longest of them i.e. 7 which lies in the second column Y 2. Since
all the components of Y2 are positive, therefore the vector Y2 will inter into the
basis YB
Now, it is required to select the vector which should leave the basis YB for this
we compute.
XB
min .y 0. i 1.2
Y12 12
13 12 1 1 X B1
i.e. min , = min 4 , 6 = 4 =
12 2 3 3 Y12
Hence the vector 1 = Y3 will leave the bases.
1 1
Thus instead of Y3 being : vector Y2 should be vector. The key
0 0
element is clearly Y21 = 3 for first iteration.
First Iteration :- Introduce the column vector Y2 and drop the column vector Y3 from
basis YB. Next convert the key element Y21 to unity by dividing its row by 3 and all
other members of column Y2 = ( 2) to zero by the application of suitable elementary
transformation.
Clearly dividing first row by 3 and subtracting first row multiplied by 2 from the
second row, we get the required result. The detail procedures are shown in the
following table.
XB Y1 Y2 Y3 Y4
R1 13 2 3 1 0
R2 12 3 2 0 1
1
Applying R1, we get
3
163
XB Y1 Y2 Y3 Y4
13 2 1
R1 1 0
3 3 3
R2 12 3 2 0 1
XB Y1 Y2 Y3 Y4
13 2 1
R1 1 0
3 3 3
5 2
R2 12 0 1
3 3
B CB XB Y1 Y2 Y3 Y4
(B1) (B2)
Y3 7 13 2 1
x3 = 1 0
3 3 3
Y4 0 10 5 2 10
x4 = 0
3 3 3 3
13 10
Z = C B XB 0 0
xj 3 3
13
=7 Cj 5 7 0 0
3
91 Cj – Zj 1 7
= 0 0
3 3 3
Incoming Outgoing
Vector vector
164
From the table, it is clear that the improved basic feasible solution is
13 10
x1 = 0, x1 = , x3 = 0, x4 =
3 3
91
Thus value of the objective function is Z =
3
As all Cj–Zj are not negative all j. So the optimum solution has not been
reached. Thus iterative process has to be repeated until an optimum solution is
finally reached.
1
It is clear from the improved table that the largest Cj – Zj is which lies is the
3
first column Y1. Since all the components of Y1 are positive, therefore the
vector Y1 will now inter the basis YB.
To select the vector which should leave the basis YB.
We compute as earlier
XB
min i
. y 0, i 1, 2
Yi1 i1
13 X B2
i.e. min ,2 = 2 =
12 Y21
5
Hence the vector 2 = Y4 leaves the basis and the element y21 = becomes the
3
key element for the second itration.
Similarly do second iteration and so on.
165
Linear Programming best outcome
model
Linear relationship
166